Programs & Examples On #Excel formula

This tag is for Microsoft Excel questions where the question or answers are specifically about the structure, syntax, or output of an Excel formula.

How to enter a formula into a cell using VBA?

You aren't building your formula right.

Worksheets("EmployeeCosts").Range("B" & var1a).Formula =  "=SUM(H5:H" & var1a & ")"

This does the same as the following lines do:

Dim myFormula As String
myFormula = "=SUM(H5:H"
myFormula = myFormula & var1a
myformula = myformula & ")"

which is what you are trying to do.

Also, you want to have the = at the beginning of the formala.

In Excel, sum all values in one column in each row where another column is a specific value

You should be able to use the IF function for that. the syntax is =IF(condition, value_if_true, value_if_false). To add an extra column with only the non-reimbursed amounts, you would use something like:

=IF(B1="No", A1, 0)

and sum that. There's probably a way to include it in a single cell below the column as well, but off the top of my head I can't think of anything simple.

Referencing value in a closed Excel workbook using INDIRECT?

I too was looking for the answer to referencing cells in a closed workbook. Here is the link to the solution (correct formula) below. I have tried it on my current project (referencing a single cell and an array of cells) and it works well with no errors. I hope it helps you.

https://www.extendoffice.com/documents/excel/4226-excel-reference-unopened-file.html

In the formula, E:\Excel file\ is the full file path of the unopened workbook, test.xlsx is the name of the workbook, Sheet2 is the sheet name which contains the cell value you need to reference from, and A:A,2,1 means the cell A2 will be referenced in the closed workbook. You can change them based on your needs.

If you want to manually select a worksheet to reference, please use this formula

=INDEX('E:\Excel file\[test.xlsx]sheetname'!A:A,2,1)

After applying this formula, you will get a Select Sheet dialog box, please select a worksheet and then click the OK button. Then the certain cell value of this worksheet will be referenced immediately.

Excel formula to display ONLY month and year?

There are a number of ways to go about this. One way would be to enter the date 8/1/2013 manually in the first cell (say A1 for example's sake) and then in B1 type the following formula (and then drag it across):

=DATE(YEAR(A1),MONTH(A1)+1,1)

Since you only want to see month and year, you can format accordingly using the different custom date formats available.

The format you're looking for is YY-Mmm.

Get content of a cell given the row and column numbers

Try =index(ARRAY, ROW, COLUMN)

where: Array: select the whole sheet Row, Column: Your row and column references

That should be easier to understand to those looking at the formula.

Excel select a value from a cell having row number calculated

You could use the INDIRECT function. This takes a string and converts it into a range

More info here

=INDIRECT("K"&A2)

But it's preferable to use INDEX as it is less volatile.

=INDEX(K:K,A2)

This returns a value or the reference to a value from within a table or range

More info here

Put either function into cell B2 and fill down.

Showing percentages above bars on Excel column graph

In Excel for Mac 2016 at least,if you place the labels in any spot on the graph and are looking to move them anywhere else (in this case above the bars), select:

Chart Design->Add Chart Element->Data Labels -> More Data Label Options

then you can grab each individual label and pull it where you would like it.

Excel formula to get week number in month (having Monday)

If week 1 always starts on the first Monday of the month try this formula for week number

=INT((6+DAY(A1+1-WEEKDAY(A1-1)))/7)

That gets the week number from the date in A1 with no intermediate calculations - if you want to use your "Monday's date" in B1 you can use this version

=INT((DAY(B1)+6)/7)

How does one sum only those rows in excel not filtered out?

If you aren't using an auto-filter (i.e. you have manually hidden rows), you will need to use the AGGREGATE function instead of SUBTOTAL.

Excel doesn't update value unless I hit Enter

It sounds like your workbook got set to Manual Calculation. You can change this to Automatic by going to Formulas > Calculation > Calculation Options > Automatic.

Location in Ribbon

Manual calculation can be useful to reduce computational load and improve responsiveness in workbooks with large amounts of formulas. The idea is that you can look at data and make changes, then choose when you want to make your computer go through the effort of calculation.

What's the quickest way to multiply multiple cells by another number?

Are you asking how to do it in excel or how to do it in a VBA application? If you just want to do it in excel, here is one way.

How to loop in excel without VBA or macros?

Going to answer this myself (correct me if I'm wrong):

It is not possible to iterate over a group of rows (like an array) in Excel without VBA installed / macros enabled.

How to compare two columns in Excel (from different sheets) and copy values from a corresponding column if the first two columns match?

As kmcamara discovered, this is exactly the kind of problem that VLOOKUP is intended to solve, and using vlookup is arguably the simplest of the alternative ways to get the job done.

In addition to the three parameters for lookup_value, table_range to be searched, and the column_index for return values, VLOOKUP takes an optional fourth argument that the Excel documentation calls the "range_lookup".

Expanding on deathApril's explanation, if this argument is set to TRUE (or 1) or omitted, the table range must be sorted in ascending order of the values in the first column of the range for the function to return what would typically be understood to be the "correct" value. Under this default behavior, the function will return a value based upon an exact match, if one is found, or an approximate match if an exact match is not found.

If the match is approximate, the value that is returned by the function will be based on the next largest value that is less than the lookup_value. For example, if "12AT8003" were missing from the table in Sheet 1, the lookup formulas for that value in Sheet 2 would return '2', since "12AT8002" is the largest value in the lookup column of the table range that is less than "12AT8003". (VLOOKUP's default behavior makes perfect sense if, for example, the goal is to look up rates in a tax table.)

However, if the fourth argument is set to FALSE (or 0), VLOOKUP returns a looked-up value only if there is an exact match, and an error value of #N/A if there is not. It is now the usual practice to wrap an exact VLOOKUP in an IFERROR function in order to catch the no-match gracefully. Prior to the introduction of IFERROR, no matches were checked with an IF function using the VLOOKUP formula once to check whether there was a match, and once to return the actual match value.

Though initially harder to master, deusxmach1na's proposed solution is a variation on a powerful set of alternatives to VLOOKUP that can be used to return values for a column or list to the left of the lookup column, expanded to handle cases where an exact match on more than one criterion is needed, or modified to incorporate OR as well as AND match conditions among multiple criteria.

Repeating kcamara's chosen solution, the VLOOKUP formula for this problem would be:

   =VLOOKUP(A1,Sheet1!A$1:B$600,2,FALSE)

Add a "sort" to a =QUERY statement in Google Spreadsheets

You can use ORDER BY clause to sort data rows by values in columns. Something like

=QUERY(responses!A1:K; "Select C, D, E where B contains '2nd Web Design' Order By C, D")

If you’d like to order by some columns descending, others ascending, you can add desc/asc, ie:

=QUERY(responses!A1:K; "Select C, D, E where B contains '2nd Web Design' Order By C desc, D")

Excel: VLOOKUP that returns true or false?

You could wrap your VLOOKUP() in an IFERROR()

Edit: before Excel 2007, use =IF(ISERROR()...)

In Excel, how do I extract last four letters of a ten letter string?

No need to use a macro. Supposing your first string is in A1.

=RIGHT(A1, 4)

Drag this down and you will get your four last characters.

Edit: To be sure, if you ever have sequences like 'ABC DEF' and want the last four LETTERS and not CHARACTERS you might want to use trimspaces()

=RIGHT(TRIMSPACES(A1), 4)

Edit: As per brettdj's suggestion, you may want to check that your string is actually 4-character long or more:

=IF(TRIMSPACES(A1)>=4, RIGHT(TRIMSPACES(A1), 4), TRIMSPACES(A1))

Simulate string split function in Excel formula

=IFERROR(LEFT(A3, FIND(" ", A3, 1)), A3)

This will firstly check if the cell contains a space, if it does it will return the first value from the space, otherwise it will return the cell value.

Edit

Just to add to the above formula, as it stands if there is no value in the cell it would return 0. If you are looking to display a message or something to tell the user it is empty you could use the following:

=IF(IFERROR(LEFT(A3, FIND(" ", A3, 1)), A3)=0, "Empty", IFERROR(LEFT(A3, FIND(" ", A3, 1)), A3))

How can I perform a reverse string search in Excel without using VBA?

=LEFT(A1,FIND(IF(
 ISERROR(
  FIND("_",A1)
 ),A1,RIGHT(A1,
  LEN(A1)-FIND("~",
   SUBSTITUTE(A1,"_","~",
    LEN(A1)-LEN(SUBSTITUTE(A1,"_",""))
   )
  )
 )
),A1,1)-2)

Excel: Can I create a Conditional Formula based on the Color of a Cell?

You can use this function (I found it here: http://excelribbon.tips.net/T010780_Colors_in_an_IF_Function.html):

Function GetFillColor(Rng As Range) As Long
    GetFillColor = Rng.Interior.ColorIndex
End Function

Here is an explanation, how to create user-defined functions: http://www.wikihow.com/Create-a-User-Defined-Function-in-Microsoft-Excel

In your worksheet, you can use the following: =GetFillColor(B5)

Simple Pivot Table to Count Unique Values

I usually sort the data by the field I need to do the distinct count of then use IF(A2=A1,0,1); you get then get a 1 in the top row of each group of IDs. Simple and doesn't take any time to calculate on large datasets.

Shortcut to Apply a Formula to an Entire Column in Excel

Select a range of cells (the entire column in this case), type in your formula, and hold down Ctrl while you press Enter. This places the formula in all selected cells.

Declaring variables in Excel Cells

You can name cells. This is done by clicking the Name Box (that thing next to the formula bar which says "A1" for example) and typing a name, such as, "myvar". Now you can use that name instead of the cell reference:

= myvar*25

Excel compare two columns and highlight duplicates

I was trying to compare A-B columns and highlight equal text, but usinng the obove fomrulas some text did not match at all. So I used form (VBA macro to compare two columns and color highlight cell differences) codes and I modified few things to adapt it to my application and find any desired column (just by clicking it). In my case, I use large and different numbers of rows on each column. Hope this helps:

Sub ABTextCompare()

Dim Report As Worksheet
Dim i, j, colNum, vMatch As Integer
Dim lastRowA, lastRowB, lastRow, lastColumn As Integer
Dim ColumnUsage As String
Dim colA, colB, colC As String
Dim A, B, C As Variant

Set Report = Excel.ActiveSheet
vMatch = 1

'Select A and B Columns to compare
On Error Resume Next
 Set A = Application.InputBox(Prompt:="Select column to compare", Title:="Column A", Type:=8)
  If A Is Nothing Then Exit Sub
colA = Split(A(1).Address(1, 0), "$")(0)
 Set B = Application.InputBox(Prompt:="Select column being searched", Title:="Column B", Type:=8)
   If A Is Nothing Then Exit Sub
  colB = Split(B(1).Address(1, 0), "$")(0)
 'Select Column to show results
 Set C = Application.InputBox("Select column  to show results", "Results", Type:=8)
    If C Is Nothing Then Exit Sub
  colC = Split(C(1).Address(1, 0), "$")(0)

'Get Last Row
lastRowA = Report.Cells.Find("", Range(colA & 1), xlFormulas, xlByRows, xlPrevious).Row - 1 ' Last row in column A
lastRowB = Report.Cells.Find("", Range(colB & 1), xlFormulas, xlByRows, xlPrevious).Row - 1 ' Last row in column B

 Application.ScreenUpdating = False
'***************************************************
For i = 2 To lastRowA
      For j = 2 To lastRowB
          If Report.Cells(i, A.Column).Value <> "" Then
              If InStr(1, Report.Cells(j, B.Column).Value, Report.Cells(i, A.Column).Value, vbTextCompare) > 0 Then
                  vMatch = vMatch + 1
                  Report.Cells(i, A.Column).Interior.ColorIndex = 35 'Light green background
                  Range(colC & 1).Value = "Items Found"
                  Report.Cells(i, A.Column).Copy Destination:=Range(colC & vMatch)
                  Exit For
              Else
                  'Do Nothing
              End If
          End If
      Next j
  Next i
If vMatch = 1 Then
    MsgBox Prompt:="No Itmes Found", Buttons:=vbInformation
End If
'***************************************************
Application.ScreenUpdating = True

End Sub

How can I sort one set of data to match another set of data in Excel?

You could also use INDEX MATCH, which is more "powerful" than vlookup. This would give you exactly what you are looking for:

enter image description here

Return Max Value of range that is determined by an Index & Match lookup

You don't need an index match formula. You can use this array formula. You have to press CTL + SHIFT + ENTER after you enter the formula.

=MAX(IF((A1:A6=A10)*(B1:B6=B10),C1:F6))

SNAPSHOT

enter image description here

How do I recognize "#VALUE!" in Excel spreadsheets?

in EXCEL 2013 i had to use IF function 2 times: 1st to identify error with ISERROR and 2nd to identify the specific type of error by ERROR.TYPE=3 in order to address this type of error. This way you can differentiate between error you want and other types.

Excel how to find values in 1 column exist in the range of values in another

Use the formula by tigeravatar:

=COUNTIF($B$2:$B$5,A2)>0 – tigeravatar Aug 28 '13 at 14:50

as conditional formatting. Highlight column A. Choose conditional formatting by forumula. Enter the formula (above) - this finds values in col B that are also in A. Choose a format (I like to use FILL and a bold color).

To find all of those values, highlight col A. Data > Filter and choose Filter by color.

In Excel how to get the left 5 characters of each cell in a specified column and put them into a new column

I find, if the data is imported, you may need to use the trim command on top of it, to get your details. =LEFT(TRIM(B2),8) In my case, I was using it to find a IP range. 10.3.44.44 with mask 255.255.255.0, so response is: 10.3.44 Kind of handy.

Error in finding last used cell in Excel with VBA

Find Last Row in a Column OR a Table Column(ListObject) by range

Finding the last row requires:

  1. Specifying several parameters (table name, column inside table relative to first column, worksheet, range).
  2. May require switching between methods. e.g. if the range is a Table (List Object) or not. Using the wrong search type will return wrong results.

This proposed solution is more general, requires only the range ,less chance of typos and is short (just calling MyLastRow function).

Sub test()
Dim rng As Range
Dim Result As Long
Set rng = Worksheets(1).Range("D4")
Result = MyLastRow(rng)
End Sub
    Function MyLastRow(FirstRow As Range) As Long
    Dim WS As Worksheet
    Dim TableName As String
    Dim ColNumber As Long
    Dim LastRow As Long
    Dim FirstColumnTable As Long
    Dim ColNumberTable As Long
    Set WS = FirstRow.Worksheet
    TableName = GetTableName(FirstRow)
    ColNumber = FirstRow.Column
    
    ''If the table (ListObject) does not start in column "A" we need to calculate the 
    ''first Column table and how many Columns from its beginning the Column is located.
    If TableName <> vbNullString Then
     FirstColumnTable = WS.ListObjects(TableName).ListColumns(1).Range.Column
     ColNumberTable = ColNumber - FirstColumnTable + 1
    End If 

    If TableName = vbNullString Then
    LastRow = WS.Cells(WS.Rows.Count, ColNumber).End(xlUp).Row
    Else
    LastRow = WS.ListObjects(TableName).ListColumns(ColNumberTable).Range.Find( _
               What:="*", SearchOrder:=xlByRows, SearchDirection:=xlPrevious).Row
    End If
    MyLastRow = LastRow
    End Function
    
    ''Get Table Name by Cell Range
    Function GetTableName(CellRange As Range) As String
        If CellRange.ListObject Is Nothing Then
            GetTableName = vbNullString
        Else
            GetTableName = CellRange.ListObject.Name
        End If
    End Function

How to compare two columns in Excel and if match, then copy the cell next to it

try this formula in column E:

=IF( AND( ISNUMBER(D2), D2=G2), H2, "")

your error is the number test, ISNUMBER( ISMATCH(D2,G:G,0) )

you do check if ismatch is-a-number, (i.e. isNumber("true") or isNumber("false"), which is not!.

I hope you understand my explanation.

Excel Date to String conversion

Here is a VBA approach:

Sub change()
    toText Sheets(1).Range("A1:F20")
End Sub

Sub toText(target As Range)
Dim cell As Range
    For Each cell In target
        cell.Value = cell.Text
        cell.NumberFormat = "@"
    Next cell
End Sub

If you are looking for a solution without programming, the Question should be moved to SuperUser.

Count if two criteria match - EXCEL formula

If youR data was in A1:C100 then:

Excel - all versions

=SUMPRODUCT(--(A1:A100="M"),--(C1:C100="Yes"))

Excel - 2007 onwards

=COUNTIFS(A1:A100,"M",C1:C100,"Yes")

Excel formula to search if all cells in a range read "True", if not, then show "False"

=COUNTIFS(1:1,FALSE)=0

This will return TRUE or FALSE (Looks for FALSE, if count isn't 0 (all True) it will be false

Check whether a cell contains a substring

Here is the formula I'm using

=IF( ISNUMBER(FIND(".",A1)), LEN(A1) - FIND(".",A1), 0 )

Format numbers in thousands (K) in Excel

[>=1000]#,##0,"K";[<=-1000]-#,##0,"K";0

teylyn's answer is great. This just adds negatives beyond -1000 following the same format.

Excel: Search for a list of strings within a particular string using array formulas?

In case others may find this useful: I found that by adding an initial empty cell to my list of search terms, a zero value will be returned instead of error.

={INDEX(SearchTerms!$A$1:$A$38,MAX(IF(ISERROR(SEARCH(SearchTerms!$A$1:$A$38,SearchCell)),0,1)*((SearchTerms!$B$1:$B$38)+1)))}

NB: Column A has the search terms, B is the row number index.

Remove leading or trailing spaces in an entire column of data

If it's the same number of characters at the beginning of the cell each time, you can use the text to columns command and select the fixed width option to chop the cell data into two columns. Then just delete the unwanted stuff in the first column.

Return values from the row above to the current row

This formula does not require a column letter reference ("A", "B", etc.). It returns the value of the cell one row above in the same column.

=INDIRECT(ADDRESS(ROW()-1,COLUMN()))

How can I count the rows with data in an Excel sheet?

With formulas, what you can do is:

  • in a new column (say col D - cell D2), add =COUNTA(A2:C2)
  • drag this formula till the end of your data (say cell D4 in our example)
  • add a last formula to sum it up (e.g in cell D5): =SUM(D2:D4)

Excel - find cell with same value in another worksheet and enter the value to the left of it

The easiest way is probably with VLOOKUP(). This will require the 2nd worksheet to have the employee number column sorted though. In newer versions of Excel, apparently sorting is no longer required.

For example, if you had a "Sheet2" with two columns - A = the employee number, B = the employee's name, and your current worksheet had employee numbers in column D and you want to fill in column E, in cell E2, you would have:

=VLOOKUP($D2, Sheet2!$A$2:$B$65535, 2, FALSE)

Then simply fill this formula down the rest of column D.

Explanation:

  • The first argument $D2 specifies the value to search for.
  • The second argument Sheet2!$A$2:$B$65535 specifies the range of cells to search in. Excel will search for the value in the first column of this range (in this case Sheet2!A2:A65535). Note I am assuming you have a header cell in row 1.
  • The third argument 2 specifies a 1-based index of the column to return from within the searched range. The value of 2 will return the second column in the range Sheet2!$A$2:$B$65535, namely the value of the B column.
  • The fourth argument FALSE says to only return exact matches.

Excel formula to reference 'CELL TO THE LEFT'

When creating your conditional formatting, set the range to which it applies to what you want (the whole sheet), then enter a relative formula (remove the $ signs) as if you were only formatting the upper-left corner.

Excel will properly apply the formatting to the rest of the cells accordingly.

In this example, starting in B1, the left cell would be A1. Just use that--no advanced formula required.


If you're looking for something more advanced, you can play around with column(), row(), and indirect(...).

Converting time stamps in excel to dates

Use this simple formula. It works.

Suppose time stamp in A2:

=DATE(YEAR(A2),MONTH(A2),DAY(A2))

How do I reference a cell within excel named range?

"Do you know if there's a way to make this work with relative selections, so that the formula can be "dragged down"/applied across several cells in the same column?"

To make such selection relative simply use ROW formula for a row number in INDEX formula and COLUMN formula for column number in INDEX formula. To make this clearer here is the example:

=INDEX(named_range,ROW(A1),COLUMN(A1))

Assuming the named range starts at A1 this formula simply indexes that range by row and column number of referenced cell and since that reference is relative it changes when you drag the the cell down or to the side, which makes it possible to create whole array of cells easily.

Conditional formatting using AND() function

I had a similar problem with a less complicated formula:

= If (x > A & x <= B) 

and found that I could Remove the AND and join the two comparisons with +

  = (x > A1) + (x <= B1)        [without all the spaces]

Hope this helps others with less complex comparisons.

Excel: last character/string match in a string

tigeravatar and Jean-François Corbett suggested to use this formula to generate the string right of the last occurrence of the "\" character

=TRIM(RIGHT(SUBSTITUTE(A1,"\",REPT(" ",LEN(A1))),LEN(A1)))

If the character used as separator is space, " ", then the formula has to be changed to:

=SUBSTITUTE(RIGHT(SUBSTITUTE(A1," ",REPT("{",LEN(A1))),LEN(A1)),"{","")

No need to mention, the "{" character can be replaced with any character that would not "normally" occur in the text to process.

If two cells match, return value from third

All you have to do is write an IF condition in the column d like this:

=IF(A1=C1;B1;" ")

After that just apply this formula to all rows above that one.

What does an exclamation mark before a cell reference mean?

If you use that forumla in the name manager you are creating a dynamic range which uses "this sheet" in place of a specific sheet.

As Jerry says, Sheet1!A1 refers to cell A1 on Sheet1. If you create a named range and omit the Sheet1 part you will reference cell A1 on the currently active sheet. (omitting the sheet reference and using it in a cell formula will error).

edit: my bad, I was using $A$1 which will lock it to the A1 cell as above, thanks pnuts :p

PivotTable to show values, not sum of values

I fear this might turn out to BE the long way round but could depend on how big your data set is – presumably more than four months for example.

Assuming your data is in ColumnA:C and has column labels in Row 1, also that Month is formatted mmm(this last for ease of sorting):

  1. Sort the data by Name then Month
  2. Enter in D2 =IF(AND(A2=A1,C2=C1),D1+1,1) (One way to deal with what is the tricky issue of multiple entries for the same person for the same month).
  3. Create a pivot table from A1:D(last occupied row no.)
  4. Say insert in F1.
  5. Layout as in screenshot.

SO12803305 example

I’m hoping this would be adequate for your needs because pivot table should automatically update (provided range is appropriate) in response to additional data with refresh. If not (you hard taskmaster), continue but beware that the following steps would need to be repeated each time the source data changes.

  1. Copy pivot table and Paste Special/Values to, say, L1.
  2. Delete top row of copied range with shift cells up.
  3. Insert new cell at L1 and shift down.
  4. Key 'Name' into L1.
  5. Filter copied range and for ColumnL, select Row Labels and numeric values.
  6. Delete contents of L2:L(last selected cell)
  7. Delete blank rows in copied range with shift cells up (may best via adding a column that counts all 12 months). Hopefully result should be as highlighted in yellow.

Happy to explain further/try again (I've not really tested this) if does not suit.

EDIT (To avoid second block of steps above and facilitate updating for source data changes)

.0. Before first step 2. add a blank row at the very top and move A2:D2 up.
.2. Adjust cell references accordingly (in D3 =IF(AND(A3=A2,C3=C2),D2+1,1).
.3. Create pivot table from A:D

.6. Overwrite Row Labels with Name.
.7. PivotTable Tools, Design, Report Layout, Show in Tabular Form and sort rows and columns A>Z.
.8. Hide Row1, ColumnG and rows and columns that show (blank).

additional example

Steps .0. and .2. in the edit are not required if the pivot table is in a different sheet from the source data (recommended).

Step .3. in the edit is a change to simplify the consequences of expanding the source data set. However introduces (blank) into pivot table that if to be hidden may need adjustment on refresh. So may be better to adjust source data range each time that changes instead: PivotTable Tools, Options, Change Data Source, Change Data Source, Select a table or range). In which case copy rather than move in .0.

Substring in excel

What about using Replace all? Just replace All on bracket to space. And comma to space. And I think you can achieve it.

How to SUM parts of a column which have same text value in different column in the same row

A PivotTable might suit, though I am not quite certain of the layout of your data:

SO19669814 example

The bold numbers (one of each pair of duplicates) need not be shown as the field does not have to be subtotalled eg:

SO19669814 second example

A formula to copy the values from a formula to another column

Copy the cell. Paste special as link. Will update with original. No formula though.

How can I color dots in a xy scatterplot according to column value?

Recently I had to do something similar and I resolved it with the code below. Hope it helps!

Sub ColorCode()
Dim i As Integer
Dim j As Integer
i = 2
j = 1

Do While ActiveSheet.Cells(i, 1) <> ""


If Cells(i, 5).Value = "RED" Then
ActiveSheet.ChartObjects("YourChartName").Chart.FullSeriesCollection(1).Points(j).MarkerForegroundColor = RGB(255, 0, 0)



Else

If Cells(i, 5).Value = "GREEN" Then
ActiveSheet.ChartObjects("YourChartName").Chart.FullSeriesCollection(1).Points(j).MarkerForegroundColor = RGB(0, 255, 0)

Else

If Cells(i, 5).Value = "GREY" Then
ActiveSheet.ChartObjects("YourChartName").Chart.FullSeriesCollection(1).Points(j).MarkerForegroundColor = RGB(192, 192, 192)

Else

If Cells(i, 5).Value = "YELLOW" Then
ActiveSheet.ChartObjects("YourChartName").Chart.FullSeriesCollection(1).Points(j).MarkerForegroundColor = RGB(255, 255, 0)

End If
End If
End If
End If

i = i + 1
j = j + 1

Loop



End Sub

How do I combine the first character of a cell with another cell in Excel?

QUESTION was: suppose T john is to be converted john T, how to change in excel?

If text "T john" is in cell A1

=CONCATENATE(RIGHT(A1,LEN(A1)-2)," ",LEFT(A1,1))

and with a nod to the & crowd

=RIGHT(A1,LEN(A1)-2)&" "&LEFT(A1,1)

takes the right part of the string excluding the first 2 characters, adds a space, adds the first character.

Refer to a cell in another worksheet by referencing the current worksheet's name?

Here is how I made monthly page in similar manner as Fernando:

  1. I wrote manually on each page number of the month and named that place as ThisMonth. Note that you can do this only before you make copies of the sheet. After copying Excel doesn't allow you to use same name, but with sheet copy it does it still. This solution works also without naming.
  2. I added number of weeks in the month to location C12. Naming is fine also.
  3. I made five weeks on every page and on fifth week I made function

      =IF(C12=5,DATE(YEAR(B48),MONTH(B48),DAY(B48)+7),"")
    

    that empties fifth week if this month has only four weeks. C12 holds the number of weeks.

  4. ...
  5. I created annual Excel, so I had 12 sheets in it: one for each month. In this example name of the sheet is "Month". Note that this solutions works also with the ODS file standard except you need to change all spaces as "_" characters.
  6. I renamed first "Month" sheet as "Month (1)" so it follows the same naming principle. You could also name it as "Month1" if you wish, but "January" would require a bit more work.
  7. Insert following function on the first day field starting sheet #2:

     =INDIRECT(CONCATENATE("'Month (",ThisMonth-1,")'!B15"))+INDIRECT(CONCATENATE("'Month (",ThisMonth-1,")'!C12"))*7
    

    So in another word, if you fill four or five weeks on the previous sheet, this calculates date correctly and continues from correct date.

How to create strings containing double quotes in Excel formulas?

will this work for macros using .Formula = "=THEFORMULAFUNCTION("STUFF")" so it would be like: will this work for macros using .Formula = "=THEFORMULAFUNCTION(CHAR(34) & STUFF & CHAR(34))"

Conditionally formatting if multiple cells are blank (no numerics throughout spreadsheet )

  1. Select columns A:H with A1 as the active cell.
  2. Open Home ? Styles ? Conditional Formatting ? New Rule.
  3. Choose Use a formula to determine which cells to format and supply one of the following formulas¹ in the Format values where this formula is true: text box.
    • To highlight the Account and Store Manager columns when one of the four dates is blank:
              =AND(LEN($A1), COLUMN()<3, COUNTBLANK($E1:$H1))
    • To highlight the Account, Store Manager and blank date columns when one of the four dates is blank:
              =AND(LEN($A1), OR(COLUMN()<3, AND(COLUMN()>4, COUNTBLANK(A1))), COUNTBLANK($E1:$H1))
  4. Click [Format] and select a cell Fill.
  5. Click [OK] to accept the formatting and then [OK] again to create the new rule. In both cases, the Applies to: will refer to =$A:$H.

Results should be similar to the following.

  Conditionally formatting if multiple cells are blank


¹ The COUNTBLANK function was introduced with Excel 2007. It will count both true blanks and zero-length strings left by formulas (e.g. "").

How to keep one variable constant with other one changing with row in excel

Use this form:

=(B0+4)/$A$0

The $ tells excel not to adjust that address while pasting the formula into new cells.

Since you are dragging across rows, you really only need to freeze the row part:

=(B0+4)/A$0

Keyboard Shortcuts

Commenters helpfully pointed out that you can toggle relative addressing for a formula in the currently selected cells with these keyboard shortcuts:

  • Windows: f4
  • Mac: CommandT

Excel: Searching for multiple terms in a cell

In addition to the answer of @teylyn, I would like to add that you can put the string of multiple search terms inside a SINGLE cell (as opposed to using a different cell for each term and then using that range as argument to SEARCH), using named ranges and the EVALUATE function as I found from this link.

For example, I put the following terms as text in a cell, $G$1:

"PRB", "utilization", "alignment", "spectrum"

Then, I defined a named range named search_terms for that cell as described in the link above and shown in the figure below:

Named range definition

In the Refers to: field I put the following:

=EVALUATE("{" & TDoc_List!$G$1 & "}")

The above EVALUATE expression is simple used to emulate the literal string

{"PRB", "utilization", "alignment", "spectrum"}

to be used as input to the SEARCH function: using a direct reference to the SINGLE cell $G$1 (augmented with the curly braces in that case) inside SEARCH does not work, hence the use of named ranges and EVALUATE.

The trick now consists in replacing the direct reference to $G$1 by the EVALUATE-augmented named range search_terms.

The formula

It really works, and shows once more how powerful Excel really is!

It really works!

Hope this helps.

Excel- compare two cell from different sheet, if true copy value from other cell

In your destination field you want to use VLOOKUP like so:

=VLOOKUP(Sheet1!A1:A100,Sheet2!A1:F100,6,FALSE)

VLOOKUP Arguments:

  1. The set fields you want to lookup.
  2. The table range you want to lookup up your value against. The first column of your defined table should be the column you want compared against your lookup field. The table range should also contain the value you want to display (Column F).
  3. This defines what field you want to display upon a match.
  4. FALSE tells VLOOKUP to do an exact match.

Sum up a column from a specific row down

This seems like the easiest (but not most robust) way to me. Simply compute the sum from row 6 to the maximum allowed row number, as specified by Excel. According to this site, the maximum is currently 1048576, so the following should work for you:

=sum(c6:c1048576)

For more robust solutions, see the other answers.

If Cell Starts with Text String... Formula

I know this is a really old post, but I found it in searching for a solution to the same problem. I don't want a nested if-statement, and Switch is apparently newer than the version of Excel I'm using. I figured out what was going wrong with my code, so I figured I'd share here in case it helps someone else.

I remembered that VLOOKUP requires the source table to be sorted alphabetically/numerically for it to work. I was initially trying to do this...

=LOOKUP(LOWER(LEFT($T$3, 1)),  {"s","l","m"}, {-1,1,0})

and it started working when I did this...

=LOOKUP(LOWER(LEFT($T$3, 1)),  {"l","m","s"}, {1,0,-1})

I was initially thinking the last value might turn out to be a default, so I wanted the zero at the last place. That doesn't seem to be the behavior anyway, so I just put the possible matches in order, and it worked.

Edit: As a final note, I see that the example in the original post has letters in alphabetical order, but I imagine the real use case might have been different if the error was happening and the letters A, B, and C were just examples.

Combining COUNT IF AND VLOOK UP EXCEL

If your are referring to two worksheets please use this formula

=COUNTIF(Worksheet2!$A$1:$A$50,Worksheet1cellA1)

In case referring to to more than two worksheets please use this formula

=COUNTIF(Worksheet2!$A$1:$A$50,Worksheet1cellA1)+=COUNTIF
(Worksheet3!$A$1:$A$50,Worksheet1cellA1)+=
               COUNTIF(Worksheet4!$A$1:$A$50,Worksheet1cellA1)

Check if an excel cell exists on another worksheet in a column - and return the contents of a different column

You can use following formulas.

For Excel 2007 or later:

=IFERROR(VLOOKUP(D3,List!A:C,3,FALSE),"No Match")

For Excel 2003:

=IF(ISERROR(MATCH(D3,List!A:A, 0)), "No Match", VLOOKUP(D3,List!A:C,3,FALSE))

Note, that

  • I'm using List!A:C in VLOOKUP and returns value from column ? 3
  • I'm using 4th argument for VLOOKUP equals to FALSE, in that case VLOOKUP will only find an exact match, and the values in the first column of List!A:C do not need to be sorted (opposite to case when you're using TRUE).

Getting unique values in Excel by using formulas only

You can also do it this way.

Create the following named ranges:

nList = the list of original values
nRow = ROW(nList)-ROW(OFFSET(nList,0,0,1,1))+1
nUnique = IF(COUNTIF(OFFSET(nList,nRow,0),nList)=0,COUNTIF(nList, "<"&nList),"")

With these 3 named ranges you can generate the ordered list of unique values with the formula below. It will be sorted in ascending order.

IFERROR(INDEX(nList,MATCH(SMALL(nUnique,ROW()-?),nUnique,0)),"")

You will need to substitute the row number of the cell just above the first element of your unique ordered list for the '?' character.

eg. If your unique ordered list begins in cell B5 then the formula will be:

IFERROR(INDEX(nList,MATCH(SMALL(nUnique,ROW()-4),nUnique,0)),"")

Set formula to a range of cells

I would update the formula in C1. Then copy the formula from C1 and paste it till C10...

Not sure about a more elegant solution

Range("C1").Formula = "=A1+B1"
Range("C1").Copy
Range("C1:C10").Pastespecial(XlPasteall)

IF - ELSE IF - ELSE Structure in Excel

Say P7 is a Cell then you can use the following Syntex to check the value of the cell and assign appropriate value to another cell based on this following nested if:

=IF(P7=0,200,IF(P7=1,100,IF(P7=2,25,IF(P7=3,10,IF((P7=4),5,0)))))

How do I convert a calendar week into a date in Excel?

If the week number is in A1 and the year in A2, you can try:

A1*7+DATE(A2,1,1)

How to Convert Excel Numeric Cell Value into Words

There is no built-in formula in excel, you have to add a vb script and permanently save it with your MS. Excel's installation as Add-In.

  1. press Alt+F11
  2. MENU: (Tool Strip) Insert Module
  3. copy and paste the below code


Option Explicit

Public Numbers As Variant, Tens As Variant

Sub SetNums()
    Numbers = Array("", "One", "Two", "Three", "Four", "Five", "Six", "Seven", "Eight", "Nine", "Ten", "Eleven", "Twelve", "Thirteen", "Fourteen", "Fifteen", "Sixteen", "Seventeen", "Eighteen", "Nineteen")
    Tens = Array("", "", "Twenty", "Thirty", "Forty", "Fifty", "Sixty", "Seventy", "Eighty", "Ninety")
End Sub

Function WordNum(MyNumber As Double) As String
    Dim DecimalPosition As Integer, ValNo As Variant, StrNo As String
    Dim NumStr As String, n As Integer, Temp1 As String, Temp2 As String
    ' This macro was written by Chris Mead - www.MeadInKent.co.uk
    If Abs(MyNumber) > 999999999 Then
        WordNum = "Value too large"
        Exit Function
    End If
    SetNums
    ' String representation of amount (excl decimals)
    NumStr = Right("000000000" & Trim(Str(Int(Abs(MyNumber)))), 9)
    ValNo = Array(0, Val(Mid(NumStr, 1, 3)), Val(Mid(NumStr, 4, 3)), Val(Mid(NumStr, 7, 3)))
    For n = 3 To 1 Step -1    'analyse the absolute number as 3 sets of 3 digits
        StrNo = Format(ValNo(n), "000")
        If ValNo(n) > 0 Then
            Temp1 = GetTens(Val(Right(StrNo, 2)))
            If Left(StrNo, 1) <> "0" Then
                Temp2 = Numbers(Val(Left(StrNo, 1))) & " hundred"
                If Temp1 <> "" Then Temp2 = Temp2 & " and "
            Else
                Temp2 = ""
            End If
            If n = 3 Then
                If Temp2 = "" And ValNo(1) + ValNo(2) > 0 Then Temp2 = "and "
                WordNum = Trim(Temp2 & Temp1)
            End If
            If n = 2 Then WordNum = Trim(Temp2 & Temp1 & " thousand " & WordNum)
            If n = 1 Then WordNum = Trim(Temp2 & Temp1 & " million " & WordNum)
        End If
    Next n
    NumStr = Trim(Str(Abs(MyNumber)))
    ' Values after the decimal place
    DecimalPosition = InStr(NumStr, ".")
    Numbers(0) = "Zero"
    If DecimalPosition > 0 And DecimalPosition < Len(NumStr) Then
        Temp1 = " point"
        For n = DecimalPosition + 1 To Len(NumStr)
            Temp1 = Temp1 & " " & Numbers(Val(Mid(NumStr, n, 1)))
        Next n
        WordNum = WordNum & Temp1
    End If
    If Len(WordNum) = 0 Or Left(WordNum, 2) = " p" Then
        WordNum = "Zero" & WordNum
    End If
End Function

Function GetTens(TensNum As Integer) As String
' Converts a number from 0 to 99 into text.
    If TensNum <= 19 Then
        GetTens = Numbers(TensNum)
    Else
        Dim MyNo As String
        MyNo = Format(TensNum, "00")
        GetTens = Tens(Val(Left(MyNo, 1))) & " " & Numbers(Val(Right(MyNo, 1)))
    End If
End Function

After this, From File Menu select Save Book ,from next menu select "Excel 97-2003 Add-In (*.xla)

It will save as Excel Add-In. that will be available till the Ms.Office Installation to that machine.

Now Open any Excel File in any Cell type =WordNum(<your numeric value or cell reference>)

you will see a Words equivalent of the numeric value.

This Snippet of code is taken from: http://en.kioskea.net/forum/affich-267274-how-to-convert-number-into-text-in-excel

Using "If cell contains #N/A" as a formula condition.

Input the following formula in C1:

=IF(ISNA(A1),B1,A1*B1)

Screenshots:

When #N/A:

enter image description here

When not #N/A:

enter image description here

Let us know if this helps.

Excel formula to remove space between words in a cell

Steps (1) Just Select your range, rows or column or array , (2) Press ctrl+H , (3 a) then in the find type a space (3 b) in the replace do not enter anything, (4)then just click on replace all..... you are done.

How to disable Excel's automatic cell reference change after copy/paste?

I found another workaround that is very simple: 1. Cut the contents 2. Paste them in the new location 3. Copy the contents that you just pasted into the new location you want. 4. Undo the Cut-Paste operation, putting the original contents back where you got them. 5. Paste the contents from the clipboard to the same location. These contents will have the original references.

It looks like a lot, but is super fast with keyboard shortcuts: 1. Ctrl-x, 2. Ctrl-v, 3. Ctrl-c, 4. Ctrl-z, 5. Ctrl-v

Excel Formula which places date/time in cell when data is entered in another cell in the same row

Another way to do this is described below.

First, turn on iterative calculations on under File - Options - Formulas - Enable Iterative Calculation. Then set maximum iterations to 1000.

After doing this, use the following formula.

=If(D55="","",IF(C55="",NOW(),C55))

Once anything is typed into cell D55 (for this example) then C55 populates today's date and/or time depending on the cell format. This date/time will not change again even if new data is entered into cell C55 so it shows the date/time that the data was entered originally.

This is a circular reference formula so you will get a warning about it every time you open the workbook. Regardless, the formula works and is easy to use anywhere you would like in the worksheet.

Conditionally formatting cells if their value equals any value of another column

I unable to comment on the top answer, but Excel actually lets you do this without adding the ugly conditional logic.

Conditional formatting is automatically applied to any input that isn't an error, so you can achieve the same effect as:

=NOT(ISERROR(MATCH(A1,$B$1:$B$1000,0)))

With this:

= MATCH(A1,$B$1:$B$1000,0)))

If the above is applied to your data, A1 will be formatted if it matches any cell in $B$1:$B$1000, as any non-match will return an error.

In excel how do I reference the current row but a specific column?

To static either a row or a column, put a $ sign in front of it. So if you were to use the formula =AVERAGE($A1,$C1) and drag it down the entire sheet, A and C would remain static while the 1 would change to the current row

If you're on Windows, you can achieve the same thing by repeatedly pressing F4 while in the formula editing bar. The first F4 press will static both (it will turn A1 into $A$1), then just the row (A$1) then just the column ($A1)

Although technically with the formulas that you have, dragging down for the entirety of the column shouldn't be a problem without putting a $ sign in front of the column. Setting the column as static would only come into play if you're dragging ACROSS columns and want to keep using the same column, and setting the row as static would be for dragging down rows but wanting to use the same row.

How can I combine multiple nested Substitute functions in Excel?

  • nesting SUBSTITUTE() in a string can be nasty, however, it's always possible to arrange it:

Screenshot formula bar

excel formula to subtract number of days from a date

Here is what worked for me (Excel 14.0 - aka MS Office Pro Plus 2010):

=DATE(YEAR(A1), MONTH(A1), DAY(A1) - 16)

This takes the date (format mm/dd/yyyy) in cell A1 and subtracts 16 days with output in format of mm/dd/yyyy.

Return empty cell from formula in Excel

Yes, it is possible.

It is possible to have a formula returning a trueblank if a condition is met. It passes the test of the ISBLANK formula. The only inconvenience is that when the condition is met, the formula will evaporate, and you will have to retype it. You can design a formula immune to self-destruction by making it return the result to the adjacent cell. Yes, it is also possible. I refer you to this solution at the end of my answer.

enter image description here

All you need is to set up a named range, say GetTrueBlank, and you will be able to use the following pattern just like in your question:

=IF(A1 = "Hello world", GetTrueBlank, A1)

Step 1. Put this code in Module of VBA.

Function Delete_UDF(rng)
    ThisWorkbook.Application.Volatile
    rng.Value = ""
End Function

Step 2. In Sheet1 in A1 cell add named range GetTrueBlank with the following formula:

=EVALUATE("Delete_UDF("&CELL("address",Sheet1!A1)&")")

enter image description here

That's it. There are no further steps. Just use self-annihilating formula. Put in the cell, say B2, the following formula:

=IF(A2=0,GetTrueBlank,A2)

The above formula in B2 will evaluate to trueblank, if you type 0 in A2.

You can download a demonstration file here.

In the example above, evaluating the formula to trueblank results in an empty cell. Checking the cell with ISBLANK formula results positively in TRUE. This is hara-kiri. The formula disappears from the cell when a condition is met. The goal is reached, although you probably might want the formula not to disappear.

You may modify the formula to return the result in the adjacent cell so that the formula will not kill itself. See how to get UDF result in the adjacent cell.

adjacent cell

I have come across the examples of getting a trueblank as a formula result revealed by The FrankensTeam here: https://sites.google.com/site/e90e50/excel-formula-to-change-the-value-of-another-cell

Excel Formula: Count cells where value is date

A bit long winded but it works for me: try this::

=SUM(IF(OR(ISBLANK(AU2), NOT(ISERR(YEAR(AU2)))),0,1)
 +IF(OR(ISBLANK(AV2), NOT(ISERR(YEAR(AV2)))),0,1))

first part of if will allow cell to be blank or if there is something in the cell it tries to convert to a year, if there is an error or there is something other than a date result = 1, do the same for each cell and sum the result

Change the fill color of a cell based on a selection from a Drop Down List in an adjacent cell

This works with me :
1- select the cells which shall be be affected by the drop down list .
2- home -> conditional formating -> new rule .
3- format only cells that contain .
4- in format only cells with ... select specific text , in formatting rule "= select Elementary from your drop down list"
if drop list in another sheet then when select Elementary we see "=Sheet3!$F$2" in the new rule , with your own sheet and cell number.
5- format -> fill -> select color -> ok.
6-ok .
do the same for each element in drop down list then you will see the magic !

Pass row number as variable in excel sheet

Assuming your row number is in B1, you can use INDIRECT:

=INDIRECT("A" & B1)

This takes a cell reference as a string (in this case, the concatenation of A and the value of B1 - 5), and returns the value at that cell.

How to remove only 0 (Zero) values from column in excel 2010

The easiest way of all is as follows: Click the office button (top left) Click "Excel Options" Click "Advanced" Scroll down to "Display options for this worksheet" Untick the box "Show a zero in cells that have zero value" Click "okay"

That's all there is to it.

:)

Extract the last substring from a cell

Try this:

=RIGHT(TRIM(A2),LEN(TRIM(A2))-FIND(" ",TRIM(A2)))

I was able to copy/paste the formula and it worked fine.

Here is a list of Excel text functions (which worked in May 2011, and but is subject to being broken the next time Microsoft changes their website). :-(

You can use a multiple-stage-nested IF() functions to handle middle names or initials, titles, etc. if you expect them. Excel formulas do not support looping, so there are some limits to what you can do.

Excel - programm cells to change colour based on another cell

Select ColumnB and as two CF formula rules apply:

Green: =AND(B1048576="X",B1="Y")

Red: =AND(B1048576="X",B1="W")

enter image description here

Using the value in a cell as a cell reference in a formula?

Use INDIRECT()

=SUM(INDIRECT(<start cell here> & ":" & <end cell here>))

How to roundup a number to the closest ten?

You can use the function MROUND(<reference cell>, <round to multiple of digit needed>).

Example:

  1. For a value A1 = 21 round to multiple of 10 it would be written as =MROUND(A1,10) for which Result = 20

  2. For a value Z4 = 55.1 round to multiple of 10 it would be written as =MROUND(Z4,10) for which Result = 60

Convert date field into text in Excel

If that is one table and have nothing to do with this - the simplest solution can be copy&paste to notepad then copy&paste back to excel :P

Sum values from multiple rows using vlookup or index/match functions

You should use Ctrl+shift+enter when using the =SUM(VLOOKUP(A9,A1:D5,{2,3,4,},FALSE)) that results in {=SUM(VLOOKUP(A9,A1:D5,{2,3,4,},FALSE))} en also works.

Ignore Duplicates and Create New List of Unique Values in Excel

So for this task First Sort your data in order from A to Z or Z to A then you can just use one simple formula as stated below:

=IF(A2=A3, "Duplicate", "Not Duplicate")

The above formula states that if column A2 data ( A is column and 2 is row number) is similar to A3 (A is Column and 3 is Row number) then it will print Duplicate else will print Not Duplicate.

Lets consider an example, Column A consists Email address in which some are duplicate, so in Column 2, I used the above stated formula which in results displayed me the 2 duplicates cells one is Row 2 and Row 6.

One you got the duplicate data just put filter on your sheet and make visible only the duplicate data and delete all the unnecessary data.

Add and Remove Views in Android Dynamically?

For Adding the Button

LinearLayout dynamicview = (LinearLayout)findViewById(R.id.buttonlayout);
LinearLayout.LayoutParams  lprams = new LinearLayout.LayoutParams(  LinearLayout.LayoutParams.WRAP_CONTENT,
LinearLayout.LayoutParams.WRAP_CONTENT);

Button btn = new Button(this);
btn.setId(count);
final int id_ = btn.getId();
btn.setText("Capture Image" + id_);
btn.setTextColor(Color.WHITE);
btn.setBackgroundColor(Color.rgb(70, 80, 90));
dynamicview.addView(btn, lprams);
btn = ((Button) findViewById(id_));
btn.setOnClickListener(this);

For removing the button

ViewGroup layout = (ViewGroup) findViewById(R.id.buttonlayout);
View command = layout.findViewById(count);
layout.removeView(command);

Check if not nil and not empty in Rails shortcut?

You can use .present? which comes included with ActiveSupport.

@city = @user.city.present?
# etc ...

You could even write it like this

def show
  %w(city state bio contact twitter mail).each do |attr|
    instance_variable_set "@#{attr}", @user[attr].present?
  end
end

It's worth noting that if you want to test if something is blank, you can use .blank? (this is the opposite of .present?)

Also, don't use foo == nil. Use foo.nil? instead.

How to map with index in Ruby?

module Enumerable
  def map_with_index(&block)
    i = 0
    self.map { |val|
      val = block.call(val, i)
      i += 1
      val
    }
  end
end

["foo", "bar"].map_with_index {|item, index| [item, index] } => [["foo", 0], ["bar", 1]]

Getting "error": "unsupported_grant_type" when trying to get a JWT by calling an OWIN OAuth secured Web Api via Postman

The response is a bit late - but in case anyone has the issue in the future...

From the screenshot above - it seems that you are adding the url data (username, password, grant_type) to the header and not to the body element.

Clicking on the body tab, and then select "x-www-form-urlencoded" radio button, there should be a key-value list below that where you can enter the request data

How to use sed to remove all double quotes within a file

Additional comment. Yes this works:

    sed 's/\"//g' infile.txt  > outfile.txt

(however with batch gnu sed, will just print to screen)

In batch scripting (GNU SED), this was needed:

    sed 's/\x22//g' infile.txt  > outfile.txt

how to add script src inside a View when using Layout

You can add the script tags like how we use in the asp.net while doing client side validations like below.

@{
    ViewBag.Title = "Index";
}

<h2>Index</h2>
<script type="text/javascript" src="~/Scripts/jquery-3.1.1.min.js"></script>
<script type="text/javascript">
    $(function () {
       //Your code
    });
</script>

NSURLErrorDomain error codes description

IN SWIFT 3. Here are the NSURLErrorDomain error codes description in a Swift 3 enum: (copied from answer above and converted what i can).

enum NSURLError: Int {
    case unknown = -1
    case cancelled = -999
    case badURL = -1000
    case timedOut = -1001
    case unsupportedURL = -1002
    case cannotFindHost = -1003
    case cannotConnectToHost = -1004
    case connectionLost = -1005
    case lookupFailed = -1006
    case HTTPTooManyRedirects = -1007
    case resourceUnavailable = -1008
    case notConnectedToInternet = -1009
    case redirectToNonExistentLocation = -1010
    case badServerResponse = -1011
    case userCancelledAuthentication = -1012
    case userAuthenticationRequired = -1013
    case zeroByteResource = -1014
    case cannotDecodeRawData = -1015
    case cannotDecodeContentData = -1016
    case cannotParseResponse = -1017
    //case NSURLErrorAppTransportSecurityRequiresSecureConnection NS_ENUM_AVAILABLE(10_11, 9_0) = -1022
    case fileDoesNotExist = -1100
    case fileIsDirectory = -1101
    case noPermissionsToReadFile = -1102
    //case NSURLErrorDataLengthExceedsMaximum NS_ENUM_AVAILABLE(10_5, 2_0) =   -1103

    // SSL errors
    case secureConnectionFailed = -1200
    case serverCertificateHasBadDate = -1201
    case serverCertificateUntrusted = -1202
    case serverCertificateHasUnknownRoot = -1203
    case serverCertificateNotYetValid = -1204
    case clientCertificateRejected = -1205
    case clientCertificateRequired = -1206
    case cannotLoadFromNetwork = -2000

    // Download and file I/O errors
    case cannotCreateFile = -3000
    case cannotOpenFile = -3001
    case cannotCloseFile = -3002
    case cannotWriteToFile = -3003
    case cannotRemoveFile = -3004
    case cannotMoveFile = -3005
    case downloadDecodingFailedMidStream = -3006
    case downloadDecodingFailedToComplete = -3007

    /*
     case NSURLErrorInternationalRoamingOff NS_ENUM_AVAILABLE(10_7, 3_0) =         -1018
     case NSURLErrorCallIsActive NS_ENUM_AVAILABLE(10_7, 3_0) =                    -1019
     case NSURLErrorDataNotAllowed NS_ENUM_AVAILABLE(10_7, 3_0) =                  -1020
     case NSURLErrorRequestBodyStreamExhausted NS_ENUM_AVAILABLE(10_7, 3_0) =      -1021

     case NSURLErrorBackgroundSessionRequiresSharedContainer NS_ENUM_AVAILABLE(10_10, 8_0) = -995
     case NSURLErrorBackgroundSessionInUseByAnotherProcess NS_ENUM_AVAILABLE(10_10, 8_0) = -996
     case NSURLErrorBackgroundSessionWasDisconnected NS_ENUM_AVAILABLE(10_10, 8_0)= -997
     */
}

Direct link to URLError.Code in the Swift github repository, which contains the up to date list of error codes being used (github link).

Image vs Bitmap class

The Bitmap class is an implementation of the Image class. The Image class is an abstract class;

The Bitmap class contains 12 constructors that construct the Bitmap object from different parameters. It can construct the Bitmap from another bitmap, and the string address of the image.

See more in this comprehensive sample.

How to run html file on localhost?

As Nora suggests, you can use the python simple server. Navigate to the folder from which you want to serve your html page, then execute python -m SimpleHTTPServer. Now you can use your web-browser and navigate to http://localhost:8000/ where your page is being served. If your page is named index.html then the server automatically loads that for you. If you want to access any other page, you'll need to browse to http://localhost:8000/{your page name}

Using textures in THREE.js

In version r75 of three.js, you should use:

var loader = new THREE.TextureLoader();
loader.load('texture.png', function ( texture ) {
  var geometry = new THREE.SphereGeometry(1000, 20, 20);
  var material = new THREE.MeshBasicMaterial({map: texture, overdraw: 0.5});
  var mesh = new THREE.Mesh(geometry, material);
  scene.add(mesh);
});

Unzip files programmatically in .net

This will do it System.IO.Compression.ZipFile.ExtractToDirectory(ZipName, ExtractToPath)

Check last modified date of file in C#

Just use File.GetLastWriteTime. There's a sample on that page showing how to use it.

response.sendRedirect() from Servlet to JSP does not seem to work

Instead of using

response.sendRedirect("/demo.jsp");

Which does a permanent redirect to an absolute URL path,

Rather use RequestDispatcher. Example:

RequestDispatcher dispatcher = request.getRequestDispatcher("demo.jsp");
dispatcher.forward(request, response);

Can I get the name of the current controller in the view?

controller_name holds the name of the controller used to serve the current view.

Select Multiple Fields from List in Linq

You can make it a KeyValuePair, so it will return a "IEnumerable<KeyValuePair<string, string>>"

So, it will be like this:

.Select(i => new KeyValuePair<string, string>(i.category_id, i.category_name )).Distinct();

Make Div Draggable using CSS

CSS is designed to describe the presentation of documents. It has a few features for changing that presentation in reaction to user interaction (primarily :hover for indicating that you are now pointing at something interactive).

Making something draggable isn't a simple matter of presentation. It is firmly in the territory of interactivity logic, which is handled by JavaScript.

What you want is not achievable.

Windows equivalent of OS X Keychain?

The "traditional" Windows equivalent would be the Protected Storage subsystem, used by IE (pre IE 7), Outlook Express, and a few other programs. I believe it's encrypted with your login password, which prevents some offline attacks, but once you're logged in, any program that wants to can read it. (See, for example, NirSoft's Protected Storage PassView.)

Windows also provides the CryptoAPI and Data Protection API that might help. Again, though, I don't think that Windows does anything to prevent processes running under the same account from seeing each other's passwords.

It looks like the book Mechanics of User Identification and Authentication provides more details on all of these.

Eclipse (via its Secure Storage feature) implements something like this, if you're interested in seeing how other software does it.

What are all codecs and formats supported by FFmpeg?

You can see the list of supported codecs in the official documentation:

Supported video codecs

Supported audio codecs

Set the selected index of a Dropdown using jQuery

JQuery code:

$("#sel_status").prop('selectedIndex',1);

Jsp Code:

Status:
<select name="sel_status"
    id="sel_status">
    <option value="1">-Status-</option>
    <option>ALL</option>
    <option>SENT</option>
    <option>RECEIVED</option>
    <option>DEACTIVE</option>
</select>

How can I make directory writable?

chmod 777 <directory>

This will give you execute/read/write privileges. You can play with the numbers to finely tune your desired permissions.

Here is the wiki with great examples.

Is False == 0 and True == 1 an implementation detail or is it guaranteed by the language?

Link to the PEP discussing the new bool type in Python 2.3: http://www.python.org/dev/peps/pep-0285/.

When converting a bool to an int, the integer value is always 0 or 1, but when converting an int to a bool, the boolean value is True for all integers except 0.

>>> int(False)
0
>>> int(True)
1
>>> bool(5)
True
>>> bool(-5)
True
>>> bool(0)
False

Flutter position stack widget in center

You can change the Positioned with Align inside a Stack:

Align(
  alignment: Alignment.bottomCenter,
  child: ... ,
),

For more info about Stack: Exploring Stack

Autocompletion of @author in Intellij

Check Enable Live Templates and leave the cursor at the position desired and click Apply then OK

enter image description here

How to write asynchronous functions for Node.js

Just passing by callbacks is not enough. You have to use settimer for example, to make function async.

Examples: Not async functions:

function a() {
  var a = 0;    
  for(i=0; i<10000000; i++) {
    a++;
  };
  b();
};

function b() {
  var a = 0;    
  for(i=0; i<10000000; i++) {
    a++;
  };    
  c();
};

function c() {
  for(i=0; i<10000000; i++) {
  };
  console.log("async finished!");
};

a();
console.log("This should be good");

If you will run above example, This should be good, will have to wait untill those functions will finish to work.

Pseudo multithread (async) functions:

function a() {
  setTimeout ( function() {
    var a = 0;  
    for(i=0; i<10000000; i++) {
      a++;
    };
    b();
  }, 0);
};

function b() {
  setTimeout ( function() {
    var a = 0;  
    for(i=0; i<10000000; i++) {
      a++;
    };  
    c();
  }, 0);
};

function c() {
  setTimeout ( function() {
    for(i=0; i<10000000; i++) {
    };
    console.log("async finished!");
  }, 0);
};

a();
console.log("This should be good");

This one will be trully async. This should be good will be writen before async finished.

Asking the user for input until they give a valid response

Use try-except to handle the error and repeat it again:

while True:
    try:
        age = int(input("Please enter your age: "))
        if age >= 18:
            print("You are able to vote in the United States!")
        else:
            print("You are not able to vote in the United States.")
    except Exception as e:
        print("please enter number")

Remove part of a string

You can use a built-in for this, strsplit:

> s = "TGAS_1121"
> s1 = unlist(strsplit(s, split='_', fixed=TRUE))[2]
> s1    
 [1] "1121"

strsplit returns both pieces of the string parsed on the split parameter as a list. That's probably not what you want, so wrap the call in unlist, then index that array so that only the second of the two elements in the vector are returned.

Finally, the fixed parameter should be set to TRUE to indicate that the split parameter is not a regular expression, but a literal matching character.

MySQL Data - Best way to implement paging?

Define OFFSET for the query. For example

page 1 - (records 01-10): offset = 0, limit=10;

page 2 - (records 11-20) offset = 10, limit =10;

and use the following query :

SELECT column FROM table LIMIT {someLimit} OFFSET {someOffset};

example for page 2:

SELECT column FROM table
LIMIT 10 OFFSET 10;

Spring @Transactional read-only propagation

Calling readOnly=false from readOnly=true doesn't work since the previous transaction continues.

In your example, the handle() method on your service layer is starting a new read-write transaction. If the handle method in turn calls service methods that annotated read-only, the read-only will take no effect as they will participate in the existing read-write transaction instead.

If it is essential for those methods to be read-only, then you can annotate them with Propagation.REQUIRES_NEW, and they will then start a new read-only transaction rather than participate in the existing read-write transaction.

Here is a worked example, CircuitStateRepository is a spring-data JPA repository.

BeanS calls a transactional=read-only Bean1, which does a lookup and calls transactional=read-write Bean2 which saves a new object.

  • Bean1 starts a read-only tx.

31 09:39:44.199 [pool-1-thread-1] DEBUG o.s.orm.jpa.JpaTransactionManager - Creating new transaction with name [nz.co.vodafone.wcim.business.Bean1.startSomething]: PROPAGATION_REQUIRED,ISOLATION_DEFAULT,readOnly; ''

  • Bean 2 pariticipates in it.

    31 09:39:44.230 [pool-1-thread-1] DEBUG o.s.orm.jpa.JpaTransactionManager - Participating in existing transaction

    Nothing is committed to the database.

Now change Bean2 @Transactional annotation to add propagation=Propagation.REQUIRES_NEW

  • Bean1 starts a read-only tx.

    31 09:31:36.418 [pool-1-thread-1] DEBUG o.s.orm.jpa.JpaTransactionManager - Creating new transaction with name [nz.co.vodafone.wcim.business.Bean1.startSomething]: PROPAGATION_REQUIRED,ISOLATION_DEFAULT,readOnly; ''

  • Bean2 starts a new read-write tx

    31 09:31:36.449 [pool-1-thread-1] DEBUG o.s.orm.jpa.JpaTransactionManager - Suspending current transaction, creating new transaction with name [nz.co.vodafone.wcim.business.Bean2.createSomething]

And the changes made by Bean2 are now committed to the database.

Here's the example, tested with spring-data, hibernate and oracle.

@Named
public class BeanS {    
    @Inject
    Bean1 bean1;

    @Scheduled(fixedRate = 20000)
    public void runSomething() {
        bean1.startSomething();
    }
}

@Named
@Transactional(readOnly = true)
public class Bean1 {    
    Logger log = LoggerFactory.getLogger(Bean1.class);

    @Inject
    private CircuitStateRepository csr;

    @Inject
    private Bean2 bean2;

    public void startSomething() {    
        Iterable<CircuitState> s = csr.findAll();
        CircuitState c = s.iterator().next();
        log.info("GOT CIRCUIT {}", c.getCircuitId());
        bean2.createSomething(c.getCircuitId());    
    }
}

@Named
@Transactional(readOnly = false)
public class Bean2 {    
    @Inject
    CircuitStateRepository csr;

    public void createSomething(String circuitId) {
        CircuitState c = new CircuitState(circuitId + "-New-" + new DateTime().toString("hhmmss"), new DateTime());

        csr.save(c);
     }
}

Convert from List into IEnumerable format

IEnumerable<Book> _Book_IE;
List<Book> _Book_List;

If it's the generic variant:

_Book_IE = _Book_List;

If you want to convert to the non-generic one:

IEnumerable ie = (IEnumerable)_Book_List;

Detect Safari browser

Note: always try to detect the specific behavior you're trying to fix, instead of targeting it with isSafari?

As a last resort, detect Safari with this regex:

var isSafari = /^((?!chrome|android).)*safari/i.test(navigator.userAgent);

It uses negative look-arounds and it excludes Chrome, Edge, and all Android browsers that include the Safari name in their user agent.

jQuery Ajax File Upload

An AJAX upload is indeed possible with XMLHttpRequest(). No iframes necessary. Upload progress can be shown.

For details see: Answer https://stackoverflow.com/a/4943774/873282 to question jQuery Upload Progress and AJAX file upload.

How to use an array list in Java?

This should do the trick:

String elem = (String)S.get(0);

Will return the first item in array.

Or

for(int i=0 ; i<S.size() ; i++){
     System.out.println(S.get(i));
}

Fatal error: Call to a member function prepare() on null

It looks like your $pdo variable is not initialized. I can't see in the code you've uploaded where you are initializing it.

Make sure you create a new PDO object in the global scope before calling the class methods. (You should declare it in the global scope because of how you implemented the methods inside the Category class).

$pdo = new PDO('mysql:host=localhost;dbname=test', $user, $pass);

Tricks to manage the available memory in an R session

Tip for dealing with objects requiring heavy intermediate calculation: When using objects that require a lot of heavy calculation and intermediate steps to create, I often find it useful to write a chunk of code with the function to create the object, and then a separate chunk of code that gives me the option either to generate and save the object as an rmd file, or load it externally from an rmd file I have already previously saved. This is especially easy to do in R Markdown using the following code-chunk structure.

```{r Create OBJECT}

COMPLICATED.FUNCTION <- function(...) { Do heavy calculations needing lots of memory;
                                        Output OBJECT; }

```
```{r Generate or load OBJECT}

LOAD <- TRUE;
#NOTE: Set LOAD to TRUE if you want to load saved file
#NOTE: Set LOAD to FALSE if you want to generate and save

if(LOAD == TRUE) { OBJECT <- readRDS(file = 'MySavedObject.rds'); } else
                 { OBJECT <- COMPLICATED.FUNCTION(x, y, z);
                             saveRDS(file = 'MySavedObject.rds', object = OBJECT); }

```

With this code structure, all I need to do is to change LOAD depending on whether I want to generate and save the object, or load it directly from an existing saved file. (Of course, I have to generate it and save it the first time, but after this I have the option of loading it.) Setting LOAD = TRUE bypasses use of my complicated function and avoids all of the heavy computation therein. This method still requires enough memory to store the object of interest, but it saves you from having to calculate it each time you run your code. For objects that require a lot of heavy calculation of intermediate steps (e.g., for calculations involving loops over large arrays) this can save a substantial amount of time and computation.

Convert list into a pandas data frame

You need convert list to numpy array and then reshape:

df = pd.DataFrame(np.array(my_list).reshape(3,3), columns = list("abc"))
print (df)
   a  b  c
0  1  2  3
1  4  5  6
2  7  8  9

Gradients on UIView and UILabels On iPhone

I realize this is an older thread, but for future reference:

As of iPhone SDK 3.0, custom gradients can be implemented very easily, without subclassing or images, by using the new CAGradientLayer:

 UIView *view = [[[UIView alloc] initWithFrame:CGRectMake(0, 0, 320, 100)] autorelease];
 CAGradientLayer *gradient = [CAGradientLayer layer];
 gradient.frame = view.bounds;
 gradient.colors = [NSArray arrayWithObjects:(id)[[UIColor blackColor] CGColor], (id)[[UIColor whiteColor] CGColor], nil];
 [view.layer insertSublayer:gradient atIndex:0];

Take a look at the CAGradientLayer docs. You can optionally specify start and end points (in case you don't want a linear gradient that goes straight from the top to the bottom), or even specific locations that map to each of the colors.

Is there an arraylist in Javascript?

Arrays are pretty flexible in JS, you can do:

var myArray = new Array();
myArray.push("string 1");
myArray.push("string 2");

Composer require runs out of memory. PHP Fatal error: Allowed memory size of 1610612736 bytes exhausted

To get the current memory_limit value, run:

php -r "echo ini_get('memory_limit').PHP_EOL;"

Try increasing the limit in your php.ini file (ex. /etc/php5/cli/php.ini for Debian-like systems):

; Use -1 for unlimited or define an explicit value like 2G
memory_limit = -1

Or, you can increase the limit with a command-line argument:

php -d memory_limit=-1 composer.phar require hwi/oauth-bundle php-http/guzzle6-adapter php-http/httplug-bundle

To get loaded php.ini files location try:

php --ini

Another quick solution:

php composer.phar COMPOSER_MEMORY_LIMIT=-1 require hwi/oauth-bundle php-http/guzzle6-adapter php-http/httplug-bundle

Append a Lists Contents to another List C#

GlobalStrings.AddRange(localStrings);

That works.

Documentation: List<T>.AddRange(IEnumerable<T>).

How to execute raw queries with Laravel 5.1?

you can run raw query like this way too.

DB::table('setting_colleges')->first();

How to iterate through a list of dictionaries in Jinja template?

**get id from dic value. I got the result.try the below code**
get_abstracts = s.get_abstracts(session_id)
    sessions = get_abstracts['sessions']
    abs = {}
    for a in get_abstracts['abstracts']:
        a_session_id = a['session_id']
        abs.setdefault(a_session_id,[]).append(a)
    authors = {}
    # print('authors')
    # print(get_abstracts['authors'])
    for au in get_abstracts['authors']: 
        # print(au)
        au_abs_id = au['abs_id']
        authors.setdefault(au_abs_id,[]).append(au)
 **In jinja template**
{% for s in sessions %}
          <h4><u>Session : {{ s.session_title}} - Hall : {{ s.session_hall}}</u></h4> 
            {% for a in abs[s.session_id] %}
            <hr>
                      <p><b>Chief Author :</b>  Dr. {{ a.full_name }}</p>  
               
                {% for au in authors[a.abs_id] %}
                      <p><b> {{ au.role }} :</b> Dr.{{ au.full_name }}</p>
                {% endfor %}
            {% endfor %}
        {% endfor %}

How to get EditText value and display it on screen through TextView?

bb.setOnClickListener(
    new View.OnClickListener()
    {
        public void onClick(View view)
        {
            String s1=tt.getText().toString();
            tv.setText(s1);
        }
    }
);

pip3: command not found

After yum install python3-pip, check the name of the installed binary. e.g.

ll /usr/bin/pip*

On my CentOS 7, it is named as pip-3 instead of pip3.

Why does Git say my master branch is "already up to date" even though it is not?

While none of these answers worked for me, I was able to fix the issue using the following command.

git fetch origin

This did a trick for me.

LaTeX beamer: way to change the bullet indentation?

Beamer just delegates responsibility for managing layout of itemize environments back to the base LaTeX packages, so there's nothing funky you need to do in Beamer itself to alter the apperaance / layout of your lists.

Since Beamer redefines itemize, item, etc., the fully proper way to manipulate things like indentation is to redefine the Beamer templates. I get the impression that you're not looking to go that far, but if that's not the case, let me know and I'll elaborate.

There are at least three ways of accomplishing your goal from within your document, without mussing about with Beamer templates.

With itemize

In the following code snippet, you can change the value of \itemindent from 0em to whatever you please, including negative values. 0em is the default item indentation.

The advantage of this method is that the list is styled normally. The disadvantage is that Beamer's redefinition of itemize and \item means that the number of paramters that can be manipulated to change the list layout is limited. It can be very hard to get the spacing right with multi-line items.

\begin{itemize}
  \setlength{\itemindent}{0em}
  \item This is a normally-indented item.
\end{itemize}

With list

In the following code snippet, the second parameter to \list is the bullet to use, and the third parameter is a list of layout parameters to change. The \leftmargin parameter adjusts the indentation of the entire list item and all of its rows; \itemindent alters the indentation of subsequent lines.

The advantage of this method is that you have all of the flexibility of lists in non-Beamer LaTeX. The disadvantage is that you have to setup the bullet style (and other visual elements) manually (or identify the right command for the template you're using). Note that if you leave the second argument empty, no bullet will be displayed and you'll save some horizontal space.

\begin{list}{$\square$}{\leftmargin=1em \itemindent=0em}
  \item This item uses the margin and indentation provided above.
\end{list}

Defining a customlist environment

The shortcomings of the list solution can be ameliorated by defining a new customlist environment that basically redefines the itemize environment from Beamer but also incorporates the \leftmargin and \itemindent (etc.) parameters. Put the following in your preamble:

\makeatletter
\newenvironment{customlist}[2]{
  \ifnum\@itemdepth >2\relax\@toodeep\else
      \advance\@itemdepth\@ne%
      \beamer@computepref\@itemdepth%
      \usebeamerfont{itemize/enumerate \beameritemnestingprefix body}%
      \usebeamercolor[fg]{itemize/enumerate \beameritemnestingprefix body}%
      \usebeamertemplate{itemize/enumerate \beameritemnestingprefix body begin}%
      \begin{list}
        {
            \usebeamertemplate{itemize \beameritemnestingprefix item}
        }
        { \leftmargin=#1 \itemindent=#2
            \def\makelabel##1{%
              {%  
                  \hss\llap{{%
                    \usebeamerfont*{itemize \beameritemnestingprefix item}%
                        \usebeamercolor[fg]{itemize \beameritemnestingprefix item}##1}}%
              }%  
            }%  
        }
  \fi
}
{
  \end{list}
  \usebeamertemplate{itemize/enumerate \beameritemnestingprefix body end}%
}
\makeatother

Now, to use an itemized list with custom indentation, you can use the following environment. The first argument is for \leftmargin and the second is for \itemindent. The default values are 2.5em and 0em respectively.

\begin{customlist}{2.5em}{0em}
   \item Any normal item can go here.
\end{customlist}

A custom bullet style can be incorporated into the customlist solution using the standard Beamer mechanism of \setbeamertemplate. (See the answers to this question on the TeX Stack Exchange for more information.)

Alternatively, the bullet style can just be modified directly within the environment, by replacing \usebeamertemplate{itemize \beameritemnestingprefix item} with whatever bullet style you'd like to use (e.g. $\square$).

How can I get a first element from a sorted list?

You have to access lists a little differently than arrays in Java. See the javadocs for the List interface for more information.

playersList.get(0)

However if you want to find the smallest element in playersList, you shouldn't sort it and then get the first element. This runs very slowly compared to just searching once through the list to find the smallest element.

For example:

int smallestIndex = 0;
for (int i = 1; i < playersList.size(); i++) {
    if (playersList.get(i) < playersList.get(smallestIndex))
        smallestIndex = i;
}

playersList.get(smallestIndex);

The above code will find the smallest element in O(n) instead of O(n log n) time.

MySQL Trigger: Delete From Table AFTER DELETE

create trigger doct_trigger
after delete on doctor
for each row
delete from patient where patient.PrimaryDoctor_SSN=doctor.SSN ;

Linq style "For Each"

Using the ToList() extension method is your best option:

someValues.ToList().ForEach(x => list.Add(x + 1));

There is no extension method in the BCL that implements ForEach directly.


Although there's no extension method in the BCL that does this, there is still an option in the System namespace... if you add Reactive Extensions to your project:

using System.Reactive.Linq;

someValues.ToObservable().Subscribe(x => list.Add(x + 1));

This has the same end result as the above use of ToList, but is (in theory) more efficient, because it streams the values directly to the delegate.

Can CSS detect the number of children an element has?

yes we can do this using nth-child like so

div:nth-child(n + 8) {
    background: red;
}

This will make the 8th div child onwards become red. Hope this helps...

Also, if someone ever says "hey, they can't be done with styled using css, use JS!!!" doubt them immediately. CSS is extremely flexible nowadays

Example :: http://jsfiddle.net/uWrLE/1/

In the example the first 7 children are blue, then 8 onwards are red...

I want to delete all bin and obj folders to force all projects to rebuild everything

A very quick and painless way is to use the rimraf npm utility, install it globally first:

> npm i rimraf -g

And then the command from your project root is quite simple (which can be saved to a script file):

projectRoot> rimraf **/bin **/obj

To optimize the desired effect you can leverage the project event targets (the one you could use is BeforeRebuild and make it run the previous command) which are specified in the docs: https://docs.microsoft.com/en-us/visualstudio/msbuild/how-to-extend-the-visual-studio-build-process?view=vs-2017

I like the rimraf utility as it is crossplat and really quick. But, you can also use the RemoveDir command in the .csproj if you decide to go with the target event option. The RemoveDir approach was well explained in another answer here by @Shaman: https://stackoverflow.com/a/22306653/1534753

How do I use tools:overrideLibrary in a build.gradle file?

Open Android Studio -> Open Manifest File

add <uses-sdk tools:overrideLibrary="android.support.v17.leanback"/> don't forget to include xmlns:tools="http://schemas.android.com/tools" too, before the <application> tag

enter image description here

Why is C so fast, and why aren't other languages as fast or faster?

Setting aside advanced optimization techniques such as hot-spot optimization, pre-compiled meta-algorithms, and various forms of parallelism, the fundamental speed of a language correlates strongly with the implicit behind-the-scenes complexity required to support the operations that would commonly be specified within inner loops.

Perhaps the most obvious is validity checking on indirect memory references -- such as checking pointers for null and checking indexes against array boundaries. Most high-level languages perform these checks implicitly, but C does not. However, this is not necessarily a fundamental limitation of these other languages -- a sufficiently clever compiler may be capable of removing these checks from the inner loops of an algorithm through some form of loop-invariant code motion.

The more fundamental advantage of C (and to a similar extent the closely related C++) is a heavy reliance on stack-based memory allocation, which is inherently fast for allocation, deallocation, and access. In C (and C++) the primary call stack can be used for allocation of primitives, arrays, and aggregates (struct/class).

While C does offer the capability to dynamically allocate memory of arbitrary size and lifetime (using the so called 'heap'), doing so is avoided by default (the stack is used instead).

Tantalizingly, it is sometimes possible to replicate the C memory allocation strategy within the runtime environments of other programming languages. This has been demonstrated by asm.js, which allows code written in C or C++ to be translated into a subset of JavaScript and run safely in a web browser environment -- with near-native speed.


As somewhat of an aside, another area where C and C++ outshine most other languages for speed is the ability to seamlessly integrate with native machine instruction sets. A notable example of this is the (compiler and platform dependent) availability of SIMD intrinsics which support the construction of custom algorithms that take advantage of the now nearly ubiquitous parallel processing hardware -- while still utilizing the data allocation abstractions provided by the language (lower-level register allocation is managed by the compiler).

Get full query string in C# ASP.NET

I have tested your example, and while Request.QueryString is not convertible to a string neither implicit nor explicit still the .ToString() method returns the correct result.

Further more when concatenating with a string using the "+" operator as in your example it will also return the correct result (because this behaves as if .ToString() was called).

As such there is nothing wrong with your code, and I would suggest that your issue was because of a typo in your code writing "Querystring" instead of "QueryString".

And this makes more sense with your error message since if the problem is that QueryString is a collection and not a string it would have to give another error message.

Difficulty with ng-model, ng-repeat, and inputs

how do something like:

<select ng-model="myModel($index+1)">

And in my inspector element be:

<select ng-model="myModel1">
...
<select ng-model="myModel2">

MySQL ORDER BY multiple column ASC and DESC

Ok, I THINK I understand what you want now, and let me clarify to confirm before the query. You want 1 record for each user. For each user, you want their BEST POINTS score record. Of the best points per user, you want the one with the best average time. Once you have all users "best" values, you want the final results sorted with best points first... Almost like ranking of a competition.

So now the query. If the above statement is accurate, you need to start with getting the best point/average time per person and assigning a "Rank" to that entry. This is easily done using MySQL @ variables. Then, just include a HAVING clause to only keep those records ranked 1 for each person. Finally apply the order by of best points and shortest average time.

select
      U.UserName,
      PreSortedPerUser.Point,
      PreSortedPerUser.Avg_Time,
      @UserRank := if( @lastUserID = PreSortedPerUser.User_ID, @UserRank +1, 1 ) FinalRank,
      @lastUserID := PreSortedPerUser.User_ID
   from
      ( select
              S.user_id,
              S.point,
              S.avg_time
           from
              Scores S
           order by
              S.user_id,
              S.point DESC,
              S.Avg_Time ) PreSortedPerUser
         JOIN Users U
            on PreSortedPerUser.user_ID = U.ID,
      ( select @lastUserID := 0,
               @UserRank := 0 ) sqlvars 
   having
      FinalRank = 1
   order by
      Point Desc,
      Avg_Time

Results as handled by SQLFiddle

Note, due to the inline @variables needed to get the answer, there are the two extra columns at the end of each row. These are just "left-over" and can be ignored in any actual output presentation you are trying to do... OR, you can wrap the entire thing above one more level to just get the few columns you want like

select 
      PQ.UserName,
      PQ.Point,
      PQ.Avg_Time
   from
      ( entire query above pasted here ) as PQ

Rails 4 Authenticity Token

Adding the following line into the form worked for me:

<%= hidden_field_tag :authenticity_token, form_authenticity_token %>

How to list all databases in the mongo shell?

For database list:

show databases
show dbs

For table/collection list:

show collections
show tables
db.getCollectionNames()

How to change the display name for LabelFor in razor in mvc3?

Decorate the model property with the DisplayName attribute.

Something like 'contains any' for Java set?

Stream::anyMatch

Since Java 8 you could use Stream::anyMatch.

setA.stream().anyMatch(setB::contains)

TypeError: You provided an invalid object where a stream was expected. You can provide an Observable, Promise, Array, or Iterable

I was forgetting to return the other observable in pipe(switchMap(

this.dataService.getPerson(personId).pipe(
  switchMap(person => {
     //this.dataService.getCompany(person.companyId); // return missing
     return this.dataService.getCompany(person.companyId);
  })
)

Rails: Default sort order for a rails model?

default_scope

This works for Rails 4+:

class Book < ActiveRecord::Base
  default_scope { order(created_at: :desc) }
end

For Rails 2.3, 3, you need this instead:

default_scope order('created_at DESC')

For Rails 2.x:

default_scope :order => 'created_at DESC'

Where created_at is the field you want the default sorting to be done on.

Note: ASC is the code to use for Ascending and DESC is for descending (desc, NOT dsc !).

scope

Once you're used to that you can also use scope:

class Book < ActiveRecord::Base
  scope :confirmed, :conditions => { :confirmed => true }
  scope :published, :conditions => { :published => true }
end

For Rails 2 you need named_scope.

:published scope gives you Book.published instead of Book.find(:published => true).

Since Rails 3 you can 'chain' those methods together by concatenating them with periods between them, so with the above scopes you can now use Book.published.confirmed.

With this method, the query is not actually executed until actual results are needed (lazy evaluation), so 7 scopes could be chained together but only resulting in 1 actual database query, to avoid performance problems from executing 7 separate queries.

You can use a passed in parameter such as a date or a user_id (something that will change at run-time and so will need that 'lazy evaluation', with a lambda, like this:

scope :recent_books, lambda 
  { |since_when| where("created_at >= ?", since_when) }
  # Note the `where` is making use of AREL syntax added in Rails 3.

Finally you can disable default scope with:

Book.with_exclusive_scope { find(:all) } 

or even better:

Book.unscoped.all

which will disable any filter (conditions) or sort (order by).

Note that the first version works in Rails2+ whereas the second (unscoped) is only for Rails3+


So ... if you're thinking, hmm, so these are just like methods then..., yup, that's exactly what these scopes are!
They are like having def self.method_name ...code... end but as always with ruby they are nice little syntactical shortcuts (or 'sugar') to make things easier for you!

In fact they are Class level methods as they operate on the 1 set of 'all' records.

Their format is changing however, with rails 4 there are deprecation warning when using #scope without passing a callable object. For example scope :red, where(color: 'red') should be changed to scope :red, -> { where(color: 'red') }.

As a side note, when used incorrectly, default_scope can be misused/abused.
This is mainly about when it gets used for actions like where's limiting (filtering) the default selection (a bad idea for a default) rather than just being used for ordering results.
For where selections, just use the regular named scopes. and add that scope on in the query, e.g. Book.all.published where published is a named scope.

In conclusion, scopes are really great and help you to push things up into the model for a 'fat model thin controller' DRYer approach.

Strange Jackson exception being thrown when serializing Hibernate object

I tried @JsonDetect and

@JsonIgnoreProperties(value = { "handler", "hibernateLazyInitializer" })

Neither of them worked for me. Using a third-party module seemed like a lot of work to me. So I just tried making a get call on any property of the lazy object before passing to jackson for serlization. The working code snippet looked something like this :

@RequestMapping(value = "/authenticate", produces = "application/json; charset=utf-8")
    @ResponseBody
    @Transactional
    public Account authenticate(Principal principal) {
        UsernamePasswordAuthenticationToken usernamePasswordAuthenticationToken = (UsernamePasswordAuthenticationToken) principal;
        LoggedInUserDetails loggedInUserDetails = (LoggedInUserDetails) usernamePasswordAuthenticationToken.getPrincipal();
        User user = userRepository.findOne(loggedInUserDetails.getUserId());
        Account account = user.getAccount();
        account.getFullName();      //Since, account is lazy giving it directly to jackson for serlization didn't worked & hence, this quick-fix.
        return account;
    }

Why AVD Manager options are not showing in Android Studio

Usually this implies some Android setup issue with the project. Go to the "Resource Manager" tab where you will be able to click on "Add Android Module" and click on import gradle files. If the import fails, you will get error messages that you can work with enter image description here

Is there a command to undo git init?

You can just delete .git. Typically:

rm -rf .git

Then, recreate as the right user.

Timeout a command in bash without unnecessary delay

I think this is precisely what you are asking for:

http://www.bashcookbook.com/bashinfo/source/bash-4.0/examples/scripts/timeout3

#!/bin/bash
#
# The Bash shell script executes a command with a time-out.
# Upon time-out expiration SIGTERM (15) is sent to the process. If the signal
# is blocked, then the subsequent SIGKILL (9) terminates it.
#
# Based on the Bash documentation example.

# Hello Chet,
# please find attached a "little easier"  :-)  to comprehend
# time-out example.  If you find it suitable, feel free to include
# anywhere: the very same logic as in the original examples/scripts, a
# little more transparent implementation to my taste.
#
# Dmitry V Golovashkin <[email protected]>

scriptName="${0##*/}"

declare -i DEFAULT_TIMEOUT=9
declare -i DEFAULT_INTERVAL=1
declare -i DEFAULT_DELAY=1

# Timeout.
declare -i timeout=DEFAULT_TIMEOUT
# Interval between checks if the process is still alive.
declare -i interval=DEFAULT_INTERVAL
# Delay between posting the SIGTERM signal and destroying the process by SIGKILL.
declare -i delay=DEFAULT_DELAY

function printUsage() {
    cat <<EOF

Synopsis
    $scriptName [-t timeout] [-i interval] [-d delay] command
    Execute a command with a time-out.
    Upon time-out expiration SIGTERM (15) is sent to the process. If SIGTERM
    signal is blocked, then the subsequent SIGKILL (9) terminates it.

    -t timeout
        Number of seconds to wait for command completion.
        Default value: $DEFAULT_TIMEOUT seconds.

    -i interval
        Interval between checks if the process is still alive.
        Positive integer, default value: $DEFAULT_INTERVAL seconds.

    -d delay
        Delay between posting the SIGTERM signal and destroying the
        process by SIGKILL. Default value: $DEFAULT_DELAY seconds.

As of today, Bash does not support floating point arithmetic (sleep does),
therefore all delay/time values must be integers.
EOF
}

# Options.
while getopts ":t:i:d:" option; do
    case "$option" in
        t) timeout=$OPTARG ;;
        i) interval=$OPTARG ;;
        d) delay=$OPTARG ;;
        *) printUsage; exit 1 ;;
    esac
done
shift $((OPTIND - 1))

# $# should be at least 1 (the command to execute), however it may be strictly
# greater than 1 if the command itself has options.
if (($# == 0 || interval <= 0)); then
    printUsage
    exit 1
fi

# kill -0 pid   Exit code indicates if a signal may be sent to $pid process.
(
    ((t = timeout))

    while ((t > 0)); do
        sleep $interval
        kill -0 $$ || exit 0
        ((t -= interval))
    done

    # Be nice, post SIGTERM first.
    # The 'exit 0' below will be executed if any preceeding command fails.
    kill -s SIGTERM $$ && kill -0 $$ || exit 0
    sleep $delay
    kill -s SIGKILL $$
) 2> /dev/null &

exec "$@"

What is code coverage and how do YOU measure it?

Just remember, having "100% code-coverage" doesn't mean everything is tested completely - while it means every line of code is tested, it doesn't mean they are tested under every (common) situation..

I would use code-coverage to highlight bits of code that I should probably write tests for. For example, if whatever code-coverage tool shows myImportantFunction() isn't executed while running my current unit-tests, they should probably be improved.

Basically, 100% code-coverage doesn't mean your code is perfect. Use it as a guide to write more comprehensive (unit-)tests.

React.js: Set innerHTML vs dangerouslySetInnerHTML

According to Dangerously Set innerHTML,

Improper use of the innerHTML can open you up to a cross-site scripting (XSS) attack. Sanitizing user input for display is notoriously error-prone, and failure to properly sanitize is one of the leading causes of web vulnerabilities on the internet.

Our design philosophy is that it should be "easy" to make things safe, and developers should explicitly state their intent when performing “unsafe” operations. The prop name dangerouslySetInnerHTML is intentionally chosen to be frightening, and the prop value (an object instead of a string) can be used to indicate sanitized data.

After fully understanding the security ramifications and properly sanitizing the data, create a new object containing only the key __html and your sanitized data as the value. Here is an example using the JSX syntax:

function createMarkup() {
    return {
       __html: 'First &middot; Second'    };
 }; 

<div dangerouslySetInnerHTML={createMarkup()} /> 

Read more about it using below link:

documentation: React DOM Elements - dangerouslySetInnerHTML.

Update values from one column in same table to another in SQL Server

This works for me

select * from stuff

update stuff
set TYPE1 = TYPE2
where TYPE1 is null;

update stuff
set TYPE1 = TYPE2
where TYPE1 ='Blank';

select * from stuff

Drawing a line/path on Google Maps

This can be done by using intents too:

  final Intent intent = new Intent(Intent.ACTION_VIEW,
    Uri.parse(
            "http://maps.google.com/maps?" +
            "saddr="+YOUR_START_LONGITUDE+","+YOUR_START_LATITUDE+"&daddr="YOUR_END_LONGITUDE+","+YOUR_END_LATITUDE));
         intent.setClassName(
          "com.google.android.apps.maps",
          "com.google.android.maps.MapsActivity");
   startActivity(intent);

How to delete images from a private docker registry?

Currently you cannot use the Registry API for that task. It only allows you to delete a repository or a specific tag.

In general, deleting a repository means, that all the tags associated to this repo are deleted.

Deleting a tag means, that the association between an image and a tag is deleted.

None of the above will delete a single image. They are left on your disk.


Workaround

For this workaround you need to have your docker images stored locally.

A workaround for your solution would be to delete all but the latest tags and thereby potentially removing the reference to the associated images. Then you can run this script to remove all images, that are not referenced by any tag or the ancestry of any used image.

Terminology (images and tags)

Consider an image graph like this where the capital letters (A, B, ...) represent short image IDs and <- means that an image is based on another image:

 A <- B <- C <- D

Now we add tags to the picture:

 A <- B <- C <- D
           |    |
           |    <version2>
           <version1>

Here, the tag <version1> references the image C and the tag <version2> references the image D.

Refining your question

In your question you said that you wanted to remove

all images but the latest

. Now, this terminology is not quite correct. You've mixed images and tags. Looking at the graph I think you would agree that the tag <version2> represents the latest version. In fact, according to this question you can have a tag that represents the latest version:

 A <- B <- C <- D
           |    |
           |    <version2>
           |    <latest>
           <version1>

Since the <latest> tag references image D I ask you: do you really want to delete all but image D? Probably not!

What happens if you delete a tag?

If you delete the tag <version1> using the Docker REST API you will get this:

 A <- B <- C <- D
                |
                <version2>
                <latest>

Remember: Docker will never delete an image! Even if it did, in this case it cannot delete an image, since the image C is part of the ancestry for the image D which is tagged.

Even if you use this script, no image will be deleted.

When an image can be deleted

Under the condition that you can control when somebody can pull or push to your registry (e.g. by disabling the REST interface). You can delete an image from an image graph if no other image is based on it and no tag refers to it.

Notice that in the following graph, the image D is not based on C but on B. Therefore, D doesn't depend on C. If you delete tag <version1> in this graph, the image C will not be used by any image and this script can remove it.

 A <- B <--------- D
      \            |
       \           <version2>
        \          <latest>
         \ <- C
              |
              <version1>

After the cleanup your image graph looks like this:

 A <- B <- D
           |
           <version2>
           <latest>

Is this what you want?

delete map[key] in go?

Copied from Go 1 release notes

In the old language, to delete the entry with key k from the map represented by m, one wrote the statement,

m[k] = value, false

This syntax was a peculiar special case, the only two-to-one assignment. It required passing a value (usually ignored) that is evaluated but discarded, plus a boolean that was nearly always the constant false. It did the job but was odd and a point of contention.

In Go 1, that syntax has gone; instead there is a new built-in function, delete. The call

delete(m, k)

will delete the map entry retrieved by the expression m[k]. There is no return value. Deleting a non-existent entry is a no-op.

Updating: Running go fix will convert expressions of the form m[k] = value, false into delete(m, k) when it is clear that the ignored value can be safely discarded from the program and false refers to the predefined boolean constant. The fix tool will flag other uses of the syntax for inspection by the programmer.

Check whether a path is valid in Python without creating a file at the path's target

try os.path.exists this will check for the path and return True if exists and False if not.

What is mapDispatchToProps?

mapStateToProps, mapDispatchToProps and connect from react-redux library provides a convenient way to access your state and dispatch function of your store. So basically connect is a higher order component, you can also think as a wrapper if this make sense for you. So every time your state is changed mapStateToProps will be called with your new state and subsequently as you props update component will run render function to render your component in browser. mapDispatchToProps also stores key-values on the props of your component, usually they take a form of a function. In such way you can trigger state change from your component onClick, onChange events.

From docs:

const TodoListComponent = ({ todos, onTodoClick }) => (
  <ul>
    {todos.map(todo =>
      <Todo
        key={todo.id}
        {...todo}
        onClick={() => onTodoClick(todo.id)}
      />
    )}
  </ul>
)

const mapStateToProps = (state) => {
  return {
    todos: getVisibleTodos(state.todos, state.visibilityFilter)
  }
}

const mapDispatchToProps = (dispatch) => {
  return {
    onTodoClick: (id) => {
      dispatch(toggleTodo(id))
    }
  }
}

function toggleTodo(index) {
  return { type: TOGGLE_TODO, index }
}

const TodoList = connect(
  mapStateToProps,
  mapDispatchToProps
)(TodoList) 

Also make sure that you are familiar with React stateless functions and Higher-Order Components

How to use a DataAdapter with stored procedure and parameter

I got it!...hehe

protected DataTable RetrieveEmployeeSubInfo(string employeeNo)
        {
            SqlCommand cmd = new SqlCommand();
            SqlDataAdapter da = new SqlDataAdapter();
            DataTable dt = new DataTable();
            try
            {
                cmd = new SqlCommand("RETRIEVE_EMPLOYEE", pl.ConnOpen());
                cmd.Parameters.Add(new SqlParameter("@EMPLOYEENO", employeeNo));
                cmd.CommandType = CommandType.StoredProcedure;
                da.SelectCommand = cmd;
                da.Fill(dt);
                dataGridView1.DataSource = dt;
            }
            catch (Exception x)
            {
                MessageBox.Show(x.GetBaseException().ToString(), "Error",
                        MessageBoxButtons.OK, MessageBoxIcon.Error);
            }
            finally
            {
                cmd.Dispose();
                pl.MySQLConn.Close();
            }
            return dt;
        }

Monitoring the Full Disclosure mailinglist

Two generic ways to do the same thing... I'm not aware of any specific open solutions to do this, but it'd be rather trivial to do.

You could write a daily or weekly cron/jenkins job to scrape the previous time period's email from the archive looking for your keyworkds/combinations. Sending a batch digest with what it finds, if anything.

But personally, I'd Setup a specific email account to subscribe to the various security lists you're interested in. Add a simple automated script to parse the new emails for various keywords or combinations of keywords, when it finds a match forward that email on to you/your team. Just be sure to keep the keywords list updated with new products you're using.

You could even do this with a gmail account and custom rules, which is what I currently do, but I have setup an internal inbox in the past with a simple python script to forward emails that were of interest.

How to remove a field completely from a MongoDB document?

Starting Mongo 4.2, it's also possible to use a slightly different syntax:

// { name: "book", tags: { words: ["abc", "123"], lat: 33, long: 22 } }
db.collection.update({}, [{ $unset: ["tags.words"] }], { many: true })
// { name: "book", tags: { lat: 33, long: 22 } }

The update method can also accept an aggregation pipeline (note the squared brackets signifying the use of an aggregation pipeline).

This means the $unset operator being used is the aggregation one (as opposed to the "query" one), whose syntax takes an array of fields.

How to write header row with csv.DictWriter?

A few options:

(1) Laboriously make an identity-mapping (i.e. do-nothing) dict out of your fieldnames so that csv.DictWriter can convert it back to a list and pass it to a csv.writer instance.

(2) The documentation mentions "the underlying writer instance" ... so just use it (example at the end).

dw.writer.writerow(dw.fieldnames)

(3) Avoid the csv.Dictwriter overhead and do it yourself with csv.writer

Writing data:

w.writerow([d[k] for k in fieldnames])

or

w.writerow([d.get(k, restval) for k in fieldnames])

Instead of the extrasaction "functionality", I'd prefer to code it myself; that way you can report ALL "extras" with the keys and values, not just the first extra key. What is a real nuisance with DictWriter is that if you've verified the keys yourself as each dict was being built, you need to remember to use extrasaction='ignore' otherwise it's going to SLOWLY (fieldnames is a list) repeat the check:

wrong_fields = [k for k in rowdict if k not in self.fieldnames]

============

>>> f = open('csvtest.csv', 'wb')
>>> import csv
>>> fns = 'foo bar zot'.split()
>>> dw = csv.DictWriter(f, fns, restval='Huh?')
# dw.writefieldnames(fns) -- no such animal
>>> dw.writerow(fns) # no such luck, it can't imagine what to do with a list
Traceback (most recent call last):
  File "<stdin>", line 1, in <module>
  File "C:\python26\lib\csv.py", line 144, in writerow
    return self.writer.writerow(self._dict_to_list(rowdict))
  File "C:\python26\lib\csv.py", line 141, in _dict_to_list
    return [rowdict.get(key, self.restval) for key in self.fieldnames]
AttributeError: 'list' object has no attribute 'get'
>>> dir(dw)
['__doc__', '__init__', '__module__', '_dict_to_list', 'extrasaction', 'fieldnam
es', 'restval', 'writer', 'writerow', 'writerows']
# eureka
>>> dw.writer.writerow(dw.fieldnames)
>>> dw.writerow({'foo':'oof'})
>>> f.close()
>>> open('csvtest.csv', 'rb').read()
'foo,bar,zot\r\noof,Huh?,Huh?\r\n'
>>>

The static keyword and its various uses in C++

When you a declare a static variable at file scope, then that variable is only available in that particular file (technically, the *translation unit, but let's not complicate this too much). For example:

a.cpp

static int x = 7;

void printax()
{
    cout << "from a.cpp: x=" << x << endl;
}

b.cpp

static int x = 9;

void printbx()
{
    cout << "from b.cpp: x=" << x << endl;
}

main.cpp:

int main(int, char **)
{
    printax(); // Will print 7
    printbx(); // Will print 9

    return 0;
}

For a local variable, static means that the variable will be zero-initialized and retain its value between calls:

unsigned int powersoftwo()
{
    static unsigned lastpow;

    if(lastpow == 0)
        lastpow = 1;
    else
        lastpow *= 2;

    return lastpow;
}

int main(int, char **)
{
    for(int i = 0; i != 10; i++)
        cout << "2^" << i << " = " << powersoftwo() << endl;
}

For class variables, it means that there is only a single instance of that variable that is shared among all members of that class. Depending on permissions, the variable can be accessed from outside the class using its fully qualified name.

class Test
{
private:
    static char *xxx;

public:
    static int yyy;

public:
    Test()
    {        
        cout << this << "The static class variable xxx is at address "
             << static_cast<void *>(xxx) << endl;
        cout << this << "The static class variable yyy is at address "
             << static_cast<void *>(&y) << endl;
    }
};

// Necessary for static class variables.
char *Test::xxx = "I'm Triple X!";
int Test::yyy = 0;

int main(int, char **)
{
    Test t1;
    Test t2;

    Test::yyy = 666;

    Test t3;
};

Marking a non-class function as static makes the function only accessible from that file and inaccessible from other files.

a.cpp

static void printfilename()
{ // this is the printfilename from a.cpp - 
  // it can't be accessed from any other file
    cout << "this is a.cpp" << endl;
}

b.cpp

static void printfilename()
{ // this is the printfilename from b.cpp - 
  // it can't be accessed from any other file
    cout << "this is b.cpp" << endl;
}

For class member functions, marking them as static means that the function doesn't need to be called on a particular instance of an object (i.e. it doesn't have a this pointer).

class Test
{
private:
    static int count;

public:
    static int GetTestCount()
    {
        return count;
    };

    Test()
    {
        cout << this << "Created an instance of Test" << endl;
        count++;
    }

    ~Test()
    {
        cout << this << "Destroyed an instance of Test" << endl;
        count--;
    }
};

int Test::count = 0;

int main(int, char **)
{
    Test *arr[10] = { NULL };

    for(int i = 0; i != 10; i++)
        arr[i] = new Test();

    cout << "There are " << Test::GetTestCount() << " instances of the Test class!" << endl;

    // now, delete them all except the first and last!
    for(int i = 1; i != 9; i++)
        delete arr[i];        

    cout << "There are " << Test::GetTestCount() << " instances of the Test class!" << endl;

    delete arr[0];

    cout << "There are " << Test::GetTestCount() << " instances of the Test class!" << endl;

    delete arr[9];

    cout << "There are " << Test::GetTestCount() << " instances of the Test class!" << endl;

    return 0;
}

Why rgb and not cmy?

The basic colours are RGB not RYB. Yes most of the softwares use the traditional RGB which can be used to mix together to form any other color i.e. RGB are the fundamental colours (as defined in Physics & Chemistry texts).

The printer user CMYK (cyan, magenta, yellow, and black) coloring as said by @jcomeau_ictx. You can view the following article to know about RGB vs CMYK: RGB Vs CMYK

A bit more information from the extract about them:

Red, Green, and Blue are "additive colors". If we combine red, green and blue light you will get white light. This is the principal behind the T.V. set in your living room and the monitor you are staring at now. Additive color, or RGB mode, is optimized for display on computer monitors and peripherals, most notably scanning devices.

Cyan, Magenta and Yellow are "subtractive colors". If we print cyan, magenta and yellow inks on white paper, they absorb the light shining on the page. Since our eyes receive no reflected light from the paper, we perceive black... in a perfect world! The printing world operates in subtractive color, or CMYK mode.

How to execute an oracle stored procedure?

I use oracle 12 and it tell me that if you need to invoke the procedure then use call keyword. In your case it should be:

begin
  call temp_proc;
end;

Getting PEAR to work on XAMPP (Apache/MySQL stack on Windows)

If you are using the portable XAMPP installation and Windows 7, and, like me have the version after they removed the XAMPP shell from the control panel none of the suggested answers here will do you much good as the packages will not install.

The problem is with the config file. I found the correct settings after a lot of trial and error.

Simply pull up a command window in the \xampp\php directory and run

pear config-set doc_dir :\xampp\php\docs\PEAR
pear config-set cfg_dir :\xampp\php\cfg
pear config-set data_dir :\xampp\php\data\PEAR
pear config-set test_dir :\xampp\php\tests
pear config-set www_dir :\xampp\php\www

you will want to replace the ':' with the actual drive letter that your portable drive is running on at the moment. Unfortunately, this needs to be done any time this drive letter changes, but it did get the module I needed installed.

Sending arrays with Intent.putExtra

You are setting the extra with an array. You are then trying to get a single int.

Your code should be:

int[] arrayB = extras.getIntArray("numbers");

LD_LIBRARY_PATH vs LIBRARY_PATH

LD_LIBRARY_PATH is searched when the program starts, LIBRARY_PATH is searched at link time.

caveat from comments:

Select SQL results grouped by weeks

I think this should do it..

Select 
ProductName,
WeekNumber,
sum(sale)
from
(
    SELECT 
    ProductName,
    DATEDIFF(week, '2011-05-30', date) AS WeekNumber,
    sale
    FROM table
)
GROUP BY
ProductName,
WeekNumber

Python: Converting from ISO-8859-1/latin1 to UTF-8

For Python 3:

bytes(apple,'iso-8859-1').decode('utf-8')

I used this for a text incorrectly encoded as iso-8859-1 (showing words like VeÅ\x99ejné) instead of utf-8. This code produces correct version Verejné.

Java : Accessing a class within a package, which is the better way?

No, it doesn't save you memory.

Also note that you don't have to import Math at all. Everything in java.lang is imported automatically.

A better example would be something like an ArrayList

import java.util.ArrayList;
....
ArrayList<String> i = new ArrayList<String>();

Note I'm importing the ArrayList specifically. I could have done

import java.util.*; 

But you generally want to avoid large wildcard imports to avoid the problem of collisions between packages.

How to quickly clear a JavaScript Object?

Assuming your object name is BASKET, simply set BASKET = "";

RegExp matching string not starting with my

You could either use a lookahead assertion like others have suggested. Or, if you just want to use basic regular expression syntax:

^(.?$|[^m].+|m[^y].*)

This matches strings that are either zero or one characters long (^.?$) and thus can not be my. Or strings with two or more characters where when the first character is not an m any more characters may follow (^[^m].+); or if the first character is a m it must not be followed by a y (^m[^y]).

How to configure nginx to enable kinda 'file browser' mode?

Just add this section to server, just before the location / {

location /your/folder/to/browse/ {
        autoindex on;
}

Create ul and li elements in javascript.

Great then. Let's create a simple function that takes an array and prints our an ordered listview/list inside a div tag.

Step 1: Let's say you have an div with "contentSectionID" id.<div id="contentSectionID"></div>

Step 2: We then create our javascript function that returns a list component and takes in an array:

function createList(spacecrafts){

var listView=document.createElement('ol');

for(var i=0;i<spacecrafts.length;i++)
{
    var listViewItem=document.createElement('li');
    listViewItem.appendChild(document.createTextNode(spacecrafts[i]));
    listView.appendChild(listViewItem);
}

return listView;
}

Step 3: Finally we select our div and create a listview in it:

document.getElementById("contentSectionID").appendChild(createList(myArr));

Execution failed for task 'app:mergeDebugResources' Crunching Cruncher....png failed

  1. Look at the folders drawable-hdpi, drawable-mdpi, drawable-xhdpi and so on.
  2. If there is a wrong bitmap file, Delete it and resave.
  3. Go to Build->Rebuild Project

pip installation /usr/local/opt/python/bin/python2.7: bad interpreter: No such file or directory

Only solution in OSX and its variant.

ln -s /usr/local/bin/python /usr/local/opt/python/bin/python2.7

handling DATETIME values 0000-00-00 00:00:00 in JDBC

To add to the other answers: If yout want the 0000-00-00 string, you can use noDatetimeStringSync=true (with the caveat of sacrificing timezone conversion).

The official MySQL bug: https://bugs.mysql.com/bug.php?id=47108.

Also, for history, JDBC used to return NULL for 0000-00-00 dates but now return an exception by default. Source

git checkout tag, git pull fails in branch

What worked for me was: git branch --set-upstream-to=origin master When I did a pull again I only got the updates from master and the warning went away.

INSERT IF NOT EXISTS ELSE UPDATE?

I think it's worth pointing out that there can be some unexpected behaviour here if you don't thoroughly understand how PRIMARY KEY and UNIQUE interact.

As an example, if you want to insert a record only if the NAME field isn't currently taken, and if it is, you want a constraint exception to fire to tell you, then INSERT OR REPLACE will not throw and exception and instead will resolve the UNIQUE constraint itself by replacing the conflicting record (the existing record with the same NAME). Gaspard's demonstrates this really well in his answer above.

If you want a constraint exception to fire, you have to use an INSERT statement, and rely on a separate UPDATE command to update the record once you know the name isn't taken.

Disable browser 'Save Password' functionality

I tested lots of solutions. Dynamic password field name, multiple password fields (invisible for fake ones), changing input type from "text" to "password", autocomplete="off", autocomplete="new-password",... but nothing solved it with recent browser.

To get rid of password remember, I finally treated the password as input field, and "blur" the text typed.

It is less "safe" than a native password field since selecting the typed text would show it as clear text, but password is not remembered. It also depends on having Javascript activated.

You will have estimate the risk of using below proposal vs password remember option from navigator.

While password remember can be managed (disbaled per site) by the user, it's fine for a personal computer, not for a "public" or shared computer.

I my case it's for a ERP running on shared computers, so I'll give it a try to my solution below.

<input style="background-color: rgb(239, 179, 196); color: black; text-shadow: none;" name="password" size="10" maxlength="30" onfocus="this.value='';this.style.color='black'; this.style.textShadow='none';" onkeypress="this.style.color='transparent'; this.style.textShadow='1px 1px 6px green';" autocomplete="off" type="text">

Clear ComboBox selected text

Try specifying the actual index of the item you want erase the text from and set it's Text equal to "".

myComboBox[this.SelectedIndex].Text = ""

or

myComboBox.selectedIndex.Text = ""

I don't remember the exact syntax but it's something along those lines.

What is the javascript filename naming convention?

The question in the link you gave talks about naming of JavaScript variables, not about file naming, so forget about that for the context in which you ask your question.

As to file naming, it is purely a matter of preference and taste. I prefer naming files with hyphens because then I don't have to reach for the shift key, as I do when dealing with camelCase file names; and because I don't have to worry about differences between Windows and Linux file names (Windows file names are case-insensitive, at least through XP).

So the answer, like so many, is "it depends" or "it's up to you."

The one rule you should follow is to be consistent in the convention you choose.

Trigger change event <select> using jquery

Give links in value of the option tag

<select size="1" name="links" onchange="window.location.href=this.value;">
   <option value="http://www.google.com">Google</option>
   <option value="http://www.yahoo.com">Yahoo</option>
</select>

UnsatisfiedDependencyException: Error creating bean with name

Considering that your package scanning is correctly set either through XML configuration or annotation based configuration.

You will need a @Repository on your ClientRepository implementation as well to allow Spring to use it in an @Autowired. Since it's not here we can only suppose that's what's missing.

As a side note, it would be cleaner to put your @Autowired/@Qualifier directly on your member if the setter method is only used for the @Autowired.

@Autowired
@Qualifier("clientRepository")
private ClientRepository clientRepository;

Lastly, you don't need the @Qualifier is there is only one class implementing the bean definition so unless you have several implementation of ClientService and ClientRepository you can remove the @Qualifier

415 Unsupported Media Type - POST json to OData service in lightswitch 2012

It looks like this issue has to do with the difference between the Content-Type and Accept headers. In HTTP, Content-Type is used in request and response payloads to convey the media type of the current payload. Accept is used in request payloads to say what media types the server may use in the response payload.

So, having a Content-Type in a request without a body (like your GET request) has no meaning. When you do a POST request, you are sending a message body, so the Content-Type does matter.

If a server is not able to process the Content-Type of the request, it will return a 415 HTTP error. (If a server is not able to satisfy any of the media types in the request Accept header, it will return a 406 error.)

In OData v3, the media type "application/json" is interpreted to mean the new JSON format ("JSON light"). If the server does not support reading JSON light, it will throw a 415 error when it sees that the incoming request is JSON light. In your payload, your request body is verbose JSON, not JSON light, so the server should be able to process your request. It just doesn't because it sees the JSON light content type.

You could fix this in one of two ways:

  1. Make the Content-Type "application/json;odata=verbose" in your POST request, or
  2. Include the DataServiceVersion header in the request and set it be less than v3. For example:

    DataServiceVersion: 2.0;
    

(Option 2 assumes that you aren't using any v3 features in your request payload.)

jquery to change style attribute of a div class

Try with

$('.handle').css({'left': '300px'});

Instead of

$('.handle').css({'style':'left: 300px'})

HTML CSS Invisible Button

Use CSS background:transparent; to your button/div.

JSFiddle

Java maximum memory on Windows XP

Keep in mind that Windows has virtual memory management and the JVM only needs memory that is contiguous in its address space. So, other programs running on the system shouldn't necessarily impact your heap size. What will get in your way are DLL's that get loaded in to your address space. Unfortunately optimizations in Windows that minimize the relocation of DLL's during linking make it more likely you'll have a fragmented address space. Things that are likely to cut in to your address space aside from the usual stuff include security software, CBT software, spyware and other forms of malware. Likely causes of the variances are different security patches, C runtime versions, etc. Device drivers and other kernel bits have their own address space (the other 2GB of the 4GB 32-bit space).

You could try going through your DLL bindings in your JVM process and look at trying to rebase your DLL's in to a more compact address space. Not fun, but if you are desperate...

Alternatively, you can just switch to 64-bit Windows and a 64-bit JVM. Despite what others have suggested, while it will chew up more RAM, you will have much more contiguous virtual address space, and allocating 2GB contiguously would be trivial.

How to add a second css class with a conditional value in razor MVC 4

You can use String.Format function to add second class based on condition:

<div class="@String.Format("details {0}", Details.Count > 0 ? "show" : "hide")">

(How) can I count the items in an enum?

Here is the best way to do it in compilation time. I have used the arg_var count answer from here.

#define PP_NARG(...) \
     PP_NARG_(__VA_ARGS__,PP_RSEQ_N())
#define PP_NARG_(...) \
         PP_ARG_N(__VA_ARGS__)

#define PP_ARG_N( \
          _1, _2, _3, _4, _5, _6, _7, _8, _9,_10, \
         _11,_12,_13,_14,_15,_16,_17,_18,_19,_20, \
         _21,_22,_23,_24,_25,_26,_27,_28,_29,_30, \
         _31,_32,_33,_34,_35,_36,_37,_38,_39,_40, \
         _41,_42,_43,_44,_45,_46,_47,_48,_49,_50, \
         _51,_52,_53,_54,_55,_56,_57,_58,_59,_60, \
         _61,_62,_63,N,...) N
#define PP_RSEQ_N() \
         63,62,61,60,                   \
         59,58,57,56,55,54,53,52,51,50, \
         49,48,47,46,45,44,43,42,41,40, \
         39,38,37,36,35,34,33,32,31,30, \
         29,28,27,26,25,24,23,22,21,20, \
         19,18,17,16,15,14,13,12,11,10, \
         9,8,7,6,5,4,3,2,1,0

#define TypedEnum(Name, ...)                                      \
struct Name {                                                     \
    enum {                                                        \
        __VA_ARGS__                                               \
    };                                                            \
    static const uint32_t Name##_MAX = PP_NARG(__VA_ARGS__);      \
}

#define Enum(Name, ...) TypedEnum(Name, __VA_ARGS__)

To declare an enum:

Enum(TestEnum, 
Enum_1= 0,
Enum_2= 1,
Enum_3= 2,
Enum_4= 4,
Enum_5= 8,
Enum_6= 16,
Enum_7= 32);

the max will be available here:

int array [TestEnum::TestEnum_MAX];
for(uint32_t fIdx = 0; fIdx < TestEnum::TestEnum_MAX; fIdx++)
{
     array [fIdx] = 0;
}

Get day of week using NSDate

There is an easier way. Just pass your string date to the following function, it will give you the day name :)

func getDayNameBy(stringDate: String) -> String
    {
        let df  = NSDateFormatter()
        df.dateFormat = "YYYY-MM-dd"
        let date = df.dateFromString(stringDate)!
        df.dateFormat = "EEEE"
        return df.stringFromDate(date);
    }

jQuery input button click event listener

First thing first, button() is a jQuery ui function to create a button widget which has nothing to do with jQuery core, it just styles the button.
So if you want to use the widget add jQuery ui's javascript and CSS files or alternatively remove it, like this:

$("#filter").click(function(){
    alert('clicked!');
});

Another thing that might have caused you the problem is if you didn't wait for the input to be rendered and wrote the code before the input. jQuery has the ready function, or it's alias $(func) which execute the callback once the DOM is ready.
Usage:

$(function(){
    $("#filter").click(function(){
        alert('clicked!');
    });
});

So even if the order is this it will work:

$(function(){
    $("#filter").click(function(){
        alert('clicked!');
    });
});

<input type="button" id="filter" name="filter" value="Filter" />

DEMO

How to check if a String contains another String in a case insensitive manner in Java?

You can use

org.apache.commons.lang3.StringUtils.containsIgnoreCase("AbBaCca", "bac");

The Apache Commons library is very useful for this sort of thing. And this particular one may be better than regular expressions as regex is always expensive in terms of performance.

How to install Android SDK Build Tools on the command line?

If you have sdkmanager installed (I'm using MAC)

run sdkmanager --list to list available packages.

If you want to install build tools, copy the preferred version from the list of packages available.

To install the preferred version run

sdkmanager "build-tools;27.0.3"

IF function with 3 conditions

You can simplify the 5 through 21 part:

=IF(E9>21,"Text1",IF(E9>4,"Text2","Text3"))

Block Comments in a Shell Script

In bash:

#!/bin/bash
echo before comment
: <<'END'
bla bla
blurfl
END
echo after comment

The ' and ' around the END delimiter are important, otherwise things inside the block like for example $(command) will be parsed and executed.

For an explanation, see this and this question.

no overload for matches delegate 'system.eventhandler'

You need to change public void klik(PaintEventArgs pea, EventArgs e) to public void klik(object sender, System.EventArgs e) because there is no Click event handler with parameters PaintEventArgs pea, EventArgs e.

Map<String, String>, how to print both the "key string" and "value string" together

Inside of your loop, you have the key, which you can use to retrieve the value from the Map:

for (String key: mss1.keySet()) {
    System.out.println(key + ": " + mss1.get(key));
}

Online SQL Query Syntax Checker

A lot of people, including me, use sqlfiddle.com to test SQL.

Listen to port via a Java socket

What do you actually want to achieve? What your code does is it tries to connect to a server located at 192.168.1.104:4000. Is this the address of a server that sends the messages (because this looks like a client-side code)? If I run fake server locally:

$ nc -l 4000

...and change socket address to localhost:4000, it will work and try to read something from nc-created server.

What you probably want is to create a ServerSocket and listen on it:

ServerSocket serverSocket = new ServerSocket(4000);
Socket socket = serverSocket.accept();

The second line will block until some other piece of software connects to your machine on port 4000. Then you can read from the returned socket. Look at this tutorial, this is actually a very broad topic (threading, protocols...)

Pointers, smart pointers or shared pointers?

the term "smart pointer" includes shared pointers, auto pointers, locking pointers and others. you meant to say auto pointer (more ambiguously known as "owning pointer"), not smart pointer.

Dumb pointers (T*) are never the best solution. They make you do explicit memory management, which is verbose, error prone, and sometimes nigh impossible. But more importantly, they don't signal your intent.

Auto pointers delete the pointee at destruction. For arrays, prefer encapsulations like vector and deque. For other objects, there's very rarely a need to store them on the heap - just use locals and object composition. Still the need for auto pointers arises with functions that return heap pointers -- such as factories and polymorphic returns.

Shared pointers delete the pointee when the last shared pointer to it is destroyed. This is useful when you want a no-brainer, open-ended storage scheme where expected lifetime and ownership can vary widely depending on the situation. Due to the need to keep an (atomic) counter, they're a bit slower than auto pointers. Some say half in jest that shared pointers are for people who can't design systems -- judge for yourself.

For an essential counterpart to shared pointers, look up weak pointers too.

How to set maximum fullscreen in vmware?

Change the resolution of your operating system running in VMware and hope it will stretch the screen when chosen the correct values

symfony2 : failed to write cache directory

If the folder is already writable so thats not the problem.

You can also just navigate to /www/projet_etienne/app/cache/ and manualy remove the folders in there (dev, dev_new, dev_old).

Make sure to SAVE a copy of those folder somewhere to put back if this doesn't fix the problem

I know this is not the way it should be done but it worked for me a couple of times now.

Add common prefix to all cells in Excel

Type this in cell B1, and copy down...

="X"&A1

This would also work:

=CONCATENATE("X",A1)

And here's one of many ways to do this in VBA (Disclaimer: I don't code in VBA very often!):

Sub AddX()
    Dim i As Long

    With ActiveSheet
    For i = 1 To .Range("A65536").End(xlUp).Row Step 1
        .Cells(i, 2).Value = "X" & Trim(Str(.Cells(i, 1).Value))
    Next i
    End With
End Sub

Entity Framework 5 Updating a Record

I really like the accepted answer. I believe there is yet another way to approach this as well. Let's say you have a very short list of properties that you wouldn't want to ever include in a View, so when updating the entity, those would be omitted. Let's say that those two fields are Password and SSN.

db.Users.Attach(updatedUser);

var entry = db.Entry(updatedUser);
entry.State = EntityState.Modified;

entry.Property(e => e.Password).IsModified = false;
entry.Property(e => e.SSN).IsModified = false;   

db.SaveChanges();   

This example allows you to essentially leave your business logic alone after adding a new field to your Users table and to your View.

Select from table by knowing only date without time (ORACLE)

DATE is a reserved keyword in Oracle, so I'm using column-name your_date instead.

If you have an index on your_date, I would use

WHERE your_date >= TO_DATE('2010-08-03', 'YYYY-MM-DD')
  AND your_date <  TO_DATE('2010-08-04', 'YYYY-MM-DD')

or BETWEEN:

WHERE your_date BETWEEN TO_DATE('2010-08-03', 'YYYY-MM-DD')
                    AND TO_DATE('2010-08-03 23:59:59', 'YYYY-MM-DD HH24:MI:SS')

If there is no index or if there are not too many records

WHERE TRUNC(your_date) = TO_DATE('2010-08-03', 'YYYY-MM-DD')

should be sufficient. TRUNC without parameter removes hours, minutes and seconds from a DATE.


If performance really matters, consider putting a Function Based Index on that column:

CREATE INDEX trunc_date_idx ON t1(TRUNC(your_date));

Split column at delimiter in data frame

The newly popular tidyr package does this with separate. It uses regular expressions so you'll have to escape the |

df <- data.frame(ID=11:13, FOO=c('a|b', 'b|c', 'x|y'))
separate(data = df, col = FOO, into = c("left", "right"), sep = "\\|")

  ID left right
1 11    a     b
2 12    b     c
3 13    x     y

though in this case the defaults are smart enough to work (it looks for non-alphanumeric characters to split on).

separate(data = df, col = FOO, into = c("left", "right"))

How to position a div in bottom right corner of a browser?

I don't have IE8 to test this out, but I'm pretty sure it should work:

<div class="screen">
   <!-- code -->
   <div class="innerdiv">
      text or other content
   </div>
</div>

and the css:

.screen{
position: relative;
}
.innerdiv {
position: absolute;
bottom: 0;
right: 0;
}

This should place the .innerdiv in the bottom-right corner of the .screen class. I hope this helps :)

Can Mockito capture arguments of a method called multiple times?

I think it should be

verify(mockBar, times(2)).doSomething(...)

Sample from mockito javadoc:

ArgumentCaptor<Person> peopleCaptor = ArgumentCaptor.forClass(Person.class);
verify(mock, times(2)).doSomething(peopleCaptor.capture());

List<Person> capturedPeople = peopleCaptor.getAllValues();
assertEquals("John", capturedPeople.get(0).getName());
assertEquals("Jane", capturedPeople.get(1).getName());

"Content is not allowed in prolog" when parsing perfectly valid XML on GAE

In my instance of the problem, the solution was to replace german umlauts (äöü) with their HTML-equivalents...

How to generate the JPA entity Metamodel?

For eclipselink, only the following dependency is sufficient to generate metamodel. Nothing else is needed.

    <dependency>
        <groupId>org.eclipse.persistence</groupId>
        <artifactId>org.eclipse.persistence.jpa.modelgen.processor</artifactId>
        <version>2.5.1</version>
        <scope>provided</scope>
    </dependency>

Android Studio suddenly cannot resolve symbols

I had a similar problem when I rebuilt an aar file and replaced the older one in my project with the new one. I went through all the solutions here and none solved my issue. I later realised that minifyEnabled had been set to true in the library project which effectively removed a lot of dead code that was not being used in the library project.

My solution was to set minifyEnabled to false in the library project, assemble the aar, copied it into my project, invalidated caches and synced the grade project and everything worked fine.

how to parse a "dd/mm/yyyy" or "dd-mm-yyyy" or "dd-mmm-yyyy" formatted date string using JavaScript or jQuery

Update

Below you've said:

Sorry, i can't predict date format before, it should be like dd-mm-yyyy or dd/mm/yyyy or dd-mmm-yyyy format finally i wanted to convert all this format to dd-MMM-yyyy format.

That completely changes the question. It'll be much more complex if you can't control the format. There is nothing built into JavaScript that will let you specify a date format. Officially, the only date format supported by JavaScript is a simplified version of ISO-8601: yyyy-mm-dd, although in practice almost all browsers also support yyyy/mm/dd as well. But other than that, you have to write the code yourself or (and this makes much more sense) use a good library. I'd probably use a library like moment.js or DateJS (although DateJS hasn't been maintained in years).


Original answer:

If the format is always dd/mm/yyyy, then this is trivial:

var parts = str.split("/");
var dt = new Date(parseInt(parts[2], 10),
                  parseInt(parts[1], 10) - 1,
                  parseInt(parts[0], 10));

split splits a string on the given delimiter. Then we use parseInt to convert the strings into numbers, and we use the new Date constructor to build a Date from those parts: The third part will be the year, the second part the month, and the first part the day. Date uses zero-based month numbers, and so we have to subtract one from the month number.

Java - Including variables within strings?

You can always use String.format(....). i.e.,

String string = String.format("A String %s %2d", aStringVar, anIntVar);

I'm not sure if that is attractive enough for you, but it can be quite handy. The syntax is the same as for printf and java.util.Formatter. I've used it much especially if I want to show tabular numeric data.

Convert DataSet to List

 List<GSTEntity.gst_jobwork_to_mfgmaster> ListToGetJwToMfData = new List<GSTEntity.gst_jobwork_to_mfgmaster>();
            DataSet getJwtMF = new DataSet();
            getJwtMF = objgst_jobwork_to_mfgmaster_BLL.GetDataJobWorkToMfg(AssesseeId, PremiseId,  Fyear,  MonthId, out webex);
            if(getJwtMF.Tables["gst_jobwork_to_mfgmaster"] != null)
            {

                ListToGetJwToMfData = (from master in getJwtMF.Tables["gst_jobwork_to_mfgmaster"].AsEnumerable() select new GSTEntity.gst_jobwork_to_mfgmaster { Partygstin = master.Field<string>("Partygstin"), Partystate =
                                       master.Field<string>("Partystate"), NatureOfTransaction = master.Field<string>("NatureOfTransaction"), ChallanNo = master.Field<string>("ChallanNo"), ChallanDate=master.Field<int>("ChallanDate"),  OtherJW_ChallanNo=master.Field<string>("OtherJW_ChallanNo"),   OtherJW_ChallanDate = master.Field<int>("OtherJW_ChallanDate"),
                    OtherJW_GSTIN=master.Field<string>("OtherJW_GSTIN"),
                    OtherJW_State = master.Field<string>("OtherJW_State"),
                    InvoiceNo = master.Field<string>("InvoiceNo"),
                    InvoiceDate=master.Field<int>("InvoiceDate"),
                    Description =master.Field<string>("Description"),
                    UQC= master.Field<string>("UQC"),
                    qty=master.Field<decimal>("qty"),
                    TaxValue=master.Field<decimal>("TaxValue"),
                    Id=master.Field<int>("Id")                        

                }).ToList();

CSS rounded corners in IE8

Rounded corners in IE8

Internet Explorer 8 (and earlier versions) doesn't support rounded corners, however there are few other solutions you may consider:

  • Use Rounded Corners Images instead (this generator is a good resource)

  • Use a jQuery Corner plugin from here

  • Use a very good script called CSS3 PIE from here (Pro's & Con's here)

  • Checkout CSS Juice from here

  • Another good script is IE-CSS3 from here

Even though CSS PIE is the most popular solution, I'm suggesting you to review all other solutions and choose what works best for your needs.

Hope it was useful. Good Luck!

Set Icon Image in Java

Use Default toolkit for this

frame.setIconImage(Toolkit.getDefaultToolkit().getImage("Icon.png"));

fatal: This operation must be run in a work tree

You repository is bare, i.e. it does not have a working tree attached to it. You can clone it locally to create a working tree for it, or you could use one of several other options to tell Git where the working tree is, e.g. the --work-tree option for single commands, or the GIT_WORK_TREE environment variable. There is also the core.worktree configuration option but it will not work in a bare repository (check the man page for what it does).

# git --work-tree=/path/to/work/tree checkout master
# GIT_WORK_TREE=/path/to/work/tree git status

How to print register values in GDB?

  • If only want check it once, info registers show registers.
  • If only want watch one register, for example, display $esp continue display esp registers in gdb command line.
  • If want watch all registers, layout regs continue show registers, with TUI mode.

sql server convert date to string MM/DD/YYYY

As of SQL Server 2012+, you can use FORMAT(value, format [, culture ])

Where the format param takes any valid standard format string or custom formatting string

Example:

SELECT FORMAT(GETDATE(), 'MM/dd/yyyy')

Further Reading:

Oracle query execution time

Use:

set serveroutput on
variable n number
exec :n := dbms_utility.get_time;
select ......
exec dbms_output.put_line( (dbms_utility.get_time-:n)/100) || ' seconds....' );

Or possibly:

SET TIMING ON;

-- do stuff

SET TIMING OFF;

...to get the hundredths of seconds that elapsed.

In either case, time elapsed can be impacted by server load/etc.

Reference:

Generate sql insert script from excel worksheet

Depending on the database, you can export to CSV and then use an import method.

MySQL - http://dev.mysql.com/doc/refman/5.1/en/load-data.html

PostgreSQL - http://www.postgresql.org/docs/8.2/static/sql-copy.html

Alternative to Intersect in MySQL

For completeness here is another method for emulating INTERSECT. Note that the IN (SELECT ...) form suggested in other answers is generally more efficient.

Generally for a table called mytable with a primary key called id:

SELECT id
FROM mytable AS a
INNER JOIN mytable AS b ON a.id = b.id
WHERE
(a.col1 = "someval")
AND
(b.col1 = "someotherval")

(Note that if you use SELECT * with this query you will get twice as many columns as are defined in mytable, this is because INNER JOIN generates a Cartesian product)

The INNER JOIN here generates every permutation of row-pairs from your table. That means every combination of rows is generated, in every possible order. The WHERE clause then filters the a side of the pair, then the b side. The result is that only rows which satisfy both conditions are returned, just like intersection two queries would do.

ActionLink htmlAttributes

@Html.ActionLink("display name", "action", "Contorller"
    new { id = 1 },Html Attribute=new {Attribute1="value"})

How can I send an inner <div> to the bottom of its parent <div>?

You may not want absolute positioning because it breaks the reflow: in some circumstances, a better solution is to make the grandparent element display:table; and the parent element display:table-cell;vertical-align:bottom;. After doing this, you should be able to give the the child elements display:inline-block; and they will automagically flow towards the bottom of the parent.

What is web.xml file and what are all things can I do with it?

What is web.xml file and what all things can I do with it ?

The /WEB-INF/web.xml file is the Web Application Deployment Descriptor of your application. This file is an XML document that defines everything about your application that a server needs to know (except the context path, which is assigned by the Application Deployer and Administrator when the application is deployed): servlets and other components like filters or listeners, initialization parameters, container-managed security constraints, resources, welcome pages, etc.

Note that reference you mentioned is pretty old (Java EE 1.4), there have been few changes in Java EE 5 and even more in Java EE 6 (which makes the web.xml "optional" and introduces Web Fragments).

Is there any configuration parameter which should be avoided like plague?

No.

Any parameters related to performance or memory usage?

No, such things are not configured at the application level but at the container level.

Security related risk due to common mis-configuration ?

Well, if you want to use container-managed security constraints and fail at configuring them properly, resources won't obviously be properly protected. Apart from that, the biggest security risks come from the code you'll deploy IMO.

Error: Cannot Start Container: stat /bin/sh: no such file or directory"

This error

docker: Error response from daemon: OCI runtime create failed: container_linux.go:348: starting container process caused "exec: \"/bin/sh\": stat /bin/sh: no such file or directory": unknown.

occurs when creating a docker image from base image eg. scratch. This is because the resulting image does not have a shell to execute the image. If your use:

ENV EXECUTABLE hello
cmd [$EXECUTABLE]

in your docker file, docker uses /bin/sh to parse the input string. and hence the error. Inspecting on the image, your will find:

$docker inspect <image-name>
"Entrypoint": [
                "/bin/sh",
                "-c",
                "[$HM_APP]"
            ]

This means that the ENTRYPOINT or CMD arguments will be parsed using /bin/sh -c. The solution that worked for me is to parse the command as a JSON array of string e.g.

cmd ["hello"]

and inspecting the image again:

"Entrypoint": [
                "hello"
            ]

This removes the dependence on /bin/sh the docker app can now execute the binary file. Example:

FROM scratch

# Environmental variables

# Copy files
ADD . /
# Home dir
WORKDIR /bin

EXPOSE 8083
ENTRYPOINT ["hospitalms"]

Hope this helps someone in future.

Reading from memory stream to string

string result = Encoding.UTF8.GetString((stream as MemoryStream).ToArray());

How to convert md5 string to normal text?

The idea of MD5 is that is a one-way hashing, so it can't be once the original value has been passed through the hashing algorithm (if at all).

You could (potentially) create a database table with a pairing of the original and the MD5 values but I guess that's highly impractical and poses a major security risk.

hash keys / values as array

function getKeys(obj){
    var keys = [];
    for (key in obj) {
        if (obj.hasOwnProperty(key)) { keys[keys.length] = key; }
    } 
    return keys;
}

Ignore parent padding

If you have a parent container with vertical padding and you want something (e.g. an image) inside that container to ignore its vertical padding you can set a negative, but equal, margin for both 'top' and 'bottom':

margin-top: -100px;
margin-bottom: -100px;

The actual value doesn't appear to matter much. Haven't tried this for horizontal paddings.

AngularJS: ng-model not binding to ng-checked for checkboxes

You don't need ng-checked when you use ng-model. If you're performing CRUD on your HTML Form, just create a model for CREATE mode that is consistent with your EDIT mode during the data-binding:

CREATE Mode: Model with default values only

$scope.dataModel = {
   isItemSelected: true,
   isApproved: true,
   somethingElse: "Your default value"
}

EDIT Mode: Model from database

$scope.dataModel = getFromDatabaseWithSameStructure()

Then whether EDIT or CREATE mode, you can consistently make use of your ng-model to sync with your database.

In C++, what is a virtual base class?

In addition to what has already been said about multiple and virtual inheritance(s), there is a very interesting article on Dr Dobb's Journal: Multiple Inheritance Considered Useful

getting only name of the class Class.getName()

The below both ways works fine.

System.out.println("The Class Name is: " + this.getClass().getName());
System.out.println("The simple Class Name is: " + this.getClass().getSimpleName());

Output as below:

The Class Name is: package.Student

The simple Class Name is: Student

How to get box-shadow on left & right sides only

I wasn't satisfied with the rounded top and bottom to the shadow present in Deefour's solution so created my own.

inset box-shadow creates a nice uniform shadow with the top and bottom cut off.

To use this effect on the sides of your element, create two pseudo elements :before and :after positioned absolutely on the sides of the original element.

_x000D_
_x000D_
div:before, div:after {
  content: " ";
  height: 100%;
  position: absolute;
  top: 0;
  width: 15px;
}
div:before {
  box-shadow: -15px 0 15px -15px inset;
  left: -15px;
}
div:after {
  box-shadow: 15px 0 15px -15px inset;
  right: -15px;
}

div {
  background: #EEEEEE;
  height: 100px;
  margin: 0 50px;
  width: 100px;
  position: relative;
}
_x000D_
<div></div>
_x000D_
_x000D_
_x000D_


Edit

Depending on your design, you may be able to use clip-path, as shown in @Luke's answer. However, note that in many cases this still results in the shadow tapering off at the top and bottom as you can see in this example:

_x000D_
_x000D_
div {
  width: 100px;
  height: 100px;
  background: #EEE;
  box-shadow: 0 0 15px 0px #000;
  clip-path: inset(0px -15px 0px -15px);
  position: relative;
  margin: 0 50px;
}
_x000D_
<div></div>
_x000D_
_x000D_
_x000D_

Run a Command Prompt command from Desktop Shortcut

The solutions turned out to be very simple.

  1. Open text edit

  2. Write the command, save as .bat.

  3. Double click the file created and the command automatically starts running in command-prompt.

enter image description here

In a bootstrap responsive page how to center a div

Here is simple CSS rules that put any div in the center

.centered {
  position: fixed;
  top: 50%;
  left: 50%;
  transform: translate(-50%, -50%);
}

https://css-tricks.com/quick-css-trick-how-to-center-an-object-exactly-in-the-center/

What are the advantages of NumPy over regular Python lists?

Here's a nice answer from the FAQ on the scipy.org website:

What advantages do NumPy arrays offer over (nested) Python lists?

Python’s lists are efficient general-purpose containers. They support (fairly) efficient insertion, deletion, appending, and concatenation, and Python’s list comprehensions make them easy to construct and manipulate. However, they have certain limitations: they don’t support “vectorized” operations like elementwise addition and multiplication, and the fact that they can contain objects of differing types mean that Python must store type information for every element, and must execute type dispatching code when operating on each element. This also means that very few list operations can be carried out by efficient C loops – each iteration would require type checks and other Python API bookkeeping.

Notepad++ add to every line

Notepad++ Add Word To Start Of Every Line

Follow this instruction to write anything at the start of every line with Notepad++

Open Notepad++,

Press Cntrl+H open the Find/Replace Dialog.

Now type ^ in the Find what textbox (Type ^ without any spaces)

Type anything(like in our example I am writing "John ") in the Replace with textbox (Write text one/more space for adding one/more space after your text in every line)

Select the Regular Expression option

Place your cursor in the first line of your file to ensure all lines are affected

Click Replace All button

enter image description here

enter image description here

Notepad++ Add Text To End Of Every Line

Follow this instruction to write anything at the end of every line with Notepad++

Open Notepad++,

Press Cntrl+H open the Find/Replace Dialog.

Now type $ in the Find what textbox (Type $ without any spaces)

Type anything(like in our example I am writing " John") in the Replace with textbox (Write one/more space text for adding one/more space before your text in every line)

Select the Regular Expression option

Place your cursor in the first line of your file to ensure all lines are affected

Click Replace All button

enter image description here

enter image description here

For all Notepadd++ Tutorials: VISIT:)

How to remove all line breaks from a string

Simple we can remove new line by using text.replace(/\n/g, " ")

_x000D_
_x000D_
const text = 'Students next year\n GO \n For Trip \n';
console.log("Original : ", text);

var removed_new_line = text.replace(/\n/g, " ");
console.log("New : ", removed_new_line);
_x000D_
_x000D_
_x000D_

fork: retry: Resource temporarily unavailable

This is commonly caused by running out of file descriptors.

There is the systems total file descriptor limit, what do you get from the command:

sysctl fs.file-nr

This returns counts of file descriptors:

<in_use> <unused_but_allocated> <maximum>

To find out what a users file descriptor limit is run the commands:

sudo su - <username>
ulimit -Hn

To find out how many file descriptors are in use by a user run the command:

sudo lsof -u <username> 2>/dev/null | wc -l

So now if you are having a system file descriptor limit issue you will need to edit your /etc/sysctl.conf file and add, or modify it it already exists, a line with fs.file-max and set it to a value large enough to deal with the number of file descriptors you need and reboot.

fs.file-max = 204708

Check table exist or not before create it in Oracle

My solution is just compilation of best ideas in thread, with a little improvement. I use both dedicated procedure (@Tomasz Borowiec) to facilitate reuse, and exception handling (@Tobias Twardon) to reduce code and to get rid of redundant table name in procedure.

DECLARE

    PROCEDURE create_table_if_doesnt_exist(
        p_create_table_query VARCHAR2
    ) IS
    BEGIN
        EXECUTE IMMEDIATE p_create_table_query;
    EXCEPTION
        WHEN OTHERS THEN
        -- suppresses "name is already being used" exception
        IF SQLCODE = -955 THEN
            NULL; 
        END IF;
    END;

BEGIN
    create_table_if_doesnt_exist('
        CREATE TABLE "MY_TABLE" (
            "ID" NUMBER(19) NOT NULL PRIMARY KEY,
            "TEXT" VARCHAR2(4000),
            "MOD_TIME" TIMESTAMP DEFAULT CURRENT_TIMESTAMP
        )
    ');
END;
/

How do you create a dropdownlist from an enum in ASP.NET MVC?

You can also use my custom HtmlHelpers in Griffin.MvcContrib. The following code:

@Html2.CheckBoxesFor(model => model.InputType) <br />
@Html2.RadioButtonsFor(model => model.InputType) <br />
@Html2.DropdownFor(model => model.InputType) <br />

Generates:

enter image description here

https://github.com/jgauffin/griffin.mvccontrib

Get an element by index in jQuery

You could skip the jquery and just use CSS style tagging:

 <ul>
 <li>India</li>
 <li>Indonesia</li>
 <li style="background-color:#343434;">China</li>
 <li>United States</li>
 <li>United Kingdom</li>
 </ul>

JAVA_HOME and PATH are set but java -version still shows the old one

In Linux Mint 18 Cinnamon be sure to check /etc/profile.d/jdk_home.sh I renamed this file to jdk_home.sh.old and now my path does not keep getting overridden and I can call java -version and see Java 9 as expected. Even though I correctly selected Java 9 in update-aternatives --config java this jdk_home.sh file kept overriding the $PATH on boot-up.

Set font-weight using Bootstrap classes

In Bootstrap 4:

class="font-weight-bold"

Or:

<strong>text</strong>

Integrate ZXing in Android Studio

Anybody facing the same issues, follow the simple steps:

Import the project android from downloaded zxing-master zip file using option Import project (Eclipse ADT, Gradle, etc.) and add the dollowing 2 lines of codes in your app level build.gradle file and and you are ready to run.

So simple, yahh...

dependencies {
        // https://mvnrepository.com/artifact/com.google.zxing/core
        compile group: 'com.google.zxing', name: 'core', version: '3.2.1'
        // https://mvnrepository.com/artifact/com.google.zxing/android-core
        compile group: 'com.google.zxing', name: 'android-core', version: '3.2.0'

    }

You can always find latest version core and android core from below links:

https://mvnrepository.com/artifact/com.google.zxing/core/3.2.1 https://mvnrepository.com/artifact/com.google.zxing/android-core/3.2.0

UPDATE (29.05.2019)

Add these dependencies instead:

dependencies {
    implementation 'com.google.zxing:core:3.4.0'
    implementation 'com.google.zxing:android-core:3.3.0'
}

How do I remove a comma off the end of a string?

If you're concatenating something in the loop, you can do it in this way too:

$coma = "";
foreach($a as $b){
    $string .= $coma.$b;
    $coma = ",";
}

"git pull" or "git merge" between master and development branches

Be careful with rebase. If you're sharing your develop branch with anybody, rebase can make a mess of things. Rebase is good only for your own local branches.

Rule of thumb, if you've pushed the branch to origin, don't use rebase. Instead, use merge.

Eclipse - Failed to load class "org.slf4j.impl.StaticLoggerBinder"

After placing the jar file in desired location, you need to add the jar file by right click on

Project --> properties --> Java Build Path --> Libraries --> Add Jar.

Is Java "pass-by-reference" or "pass-by-value"?

public class Test {

    static class Dog {
        String name;

        @Override
        public int hashCode() {
            final int prime = 31;
            int result = 1;
            result = prime * result + ((name == null) ? 0 : name.hashCode());
            return result;
        }

        @Override
        public boolean equals(Object obj) {
            if (this == obj)
                return true;
            if (obj == null)
                return false;
            if (getClass() != obj.getClass())
                return false;
            Dog other = (Dog) obj;
            if (name == null) {
                if (other.name != null)
                    return false;
            } else if (!name.equals(other.name))
                return false;
            return true;
        }

        public String getName() {
            return name;
        }

        public void setName(String nb) {
            this.name = nb;
        }

        Dog(String sd) {
            this.name = sd;
        }
    }
    /**
     * 
     * @param args
     */
    public static void main(String[] args) {
        Dog aDog = new Dog("Max");

        // we pass the object to foo
        foo(aDog);
        Dog oldDog = aDog;

        System.out.println(" 1: " + aDog.getName().equals("Max")); // false
        System.out.println(" 2 " + aDog.getName().equals("huahua")); // false
        System.out.println(" 3 " + aDog.getName().equals("moron")); // true
        System.out.println(" 4 " + " " + (aDog == oldDog)); // true

        // part2
        Dog aDog1 = new Dog("Max");

        foo(aDog1, 5);
        Dog oldDog1 = aDog;

        System.out.println(" 5 : " + aDog1.getName().equals("huahua")); // true
        System.out.println(" part2 : " + (aDog1 == oldDog1)); // false

        Dog oldDog2 = foo(aDog1, 5, 6);
        System.out.println(" 6 " + (aDog1 == oldDog2)); // true
        System.out.println(" 7 " + (aDog1 == oldDog)); // false
        System.out.println(" 8 " + (aDog == oldDog2)); // false
    }

    /**
     * 
     * @param d
     */
    public static void foo(Dog d) {
        System.out.println(d.getName().equals("Max")); // true

        d.setName("moron");

        d = new Dog("huahua");
        System.out.println(" -:-  " + d.getName().equals("huahua")); // true
    }

    /**
     * 
     * @param d
     * @param a
     */
    public static void foo(Dog d, int a) {
        d.getName().equals("Max"); // true

        d.setName("huahua");
    }

    /**
     * 
     * @param d
     * @param a
     * @param b
     * @return
     */
    public static Dog foo(Dog d, int a, int b) {
        d.getName().equals("Max"); // true
        d.setName("huahua");
        return d;
    }
}

The sample code to demonstrate the impact of changes to the object at different functions .

How to vertically align into the center of the content of a div with defined width/height?

margin: all_four_margin

by providing 50% to all_four_margin will place the element at the center

style="margin: 50%"

you can apply it for following too

margin: top right bottom left

margin: top right&left bottom

margin: top&bottom right&left

by giving appropriate % we get the element wherever we want.

Add new attribute (element) to JSON object using JavaScript

You can also dynamically add attributes with variables directly in an object literal.

const amountAttribute = 'amount';
const foo = {
                [amountAttribute]: 1
            };
foo[amountAttribute + "__more"] = 2;

Results in:

{
    amount: 1, 
    amount__more: 2
}

What is the default value for enum variable?

It is whatever member of the enumeration represents the value 0. Specifically, from the documentation:

The default value of an enum E is the value produced by the expression (E)0.

As an example, take the following enum:

enum E
{
    Foo, Bar, Baz, Quux
}

Without overriding the default values, printing default(E) returns Foo since it's the first-occurring element.

However, it is not always the case that 0 of an enum is represented by the first member. For example, if you do this:

enum F
{
    // Give each element a custom value
    Foo = 1, Bar = 2, Baz = 3, Quux = 0
}

Printing default(F) will give you Quux, not Foo.

If none of the elements in an enum G correspond to 0:

enum G
{
    Foo = 1, Bar = 2, Baz = 3, Quux = 4
}

default(G) returns literally 0, although its type remains as G (as quoted by the docs above, a cast to the given enum type).

Are string.Equals() and == operator really same?

Two differences:

  • Equals is polymorphic (i.e. it can be overridden, and the implementation used will depend on the execution-time type of the target object), whereas the implementation of == used is determined based on the compile-time types of the objects:

    // Avoid getting confused by interning
    object x = new StringBuilder("hello").ToString();
    object y = new StringBuilder("hello").ToString();
    if (x.Equals(y)) // Yes
    
    // The compiler doesn't know to call ==(string, string) so it generates
    // a reference comparision instead
    if (x == y) // No
    
    string xs = (string) x;
    string ys = (string) y;
    
    // Now *this* will call ==(string, string), comparing values appropriately
    if (xs == ys) // Yes
    
  • Equals will go bang if you call it on null, == won't

    string x = null;
    string y = null;
    
    if (x.Equals(y)) // Bang
    
    if (x == y) // Yes
    

Note that you can avoid the latter being a problem using object.Equals:

if (object.Equals(x, y)) // Fine even if x or y is null

JavaScript: get code to run every minute

Using setInterval:

setInterval(function() {
    // your code goes here...
}, 60 * 1000); // 60 * 1000 milsec

The function returns an id you can clear your interval with clearInterval:

var timerID = setInterval(function() {
    // your code goes here...
}, 60 * 1000); 

clearInterval(timerID); // The setInterval it cleared and doesn't run anymore.

A "sister" function is setTimeout/clearTimeout look them up.


If you want to run a function on page init and then 60 seconds after, 120 sec after, ...:

function fn60sec() {
    // runs every 60 sec and runs on init.
}
fn60sec();
setInterval(fn60sec, 60*1000);

Use ffmpeg to add text subtitles

ffmpeg -i infile.mp4 -i infile.srt -c copy -c:s mov_text outfile.mp4

-vf subtitles=infile.srt will not work with -c copy


The order of -c copy -c:s mov_text is important. You are telling FFmpeg:

  1. Video: copy, Audio: copy, Subtitle: copy
  2. Subtitle: mov_text

If you reverse them, you are telling FFmpeg:

  1. Subtitle: mov_text
  2. Video: copy, Audio: copy, Subtitle: copy

Alternatively you could just use -c:v copy -c:a copy -c:s mov_text in any order.

How to get the cursor to change to the hand when hovering a <button> tag

Just add this style:

cursor: pointer;

The reason it's not happening by default is because most browsers reserve the pointer for links only (and maybe a couple other things I'm forgetting, but typically not <button>s).

More on the cursor property: https://developer.mozilla.org/en/CSS/cursor

I usually apply this to <button> and <label> by default.

NOTE: I just caught this:

the button tags have an id of #more

It's very important that each element has it's own unique id, you cannot have duplicates. Use the class attribute instead, and change your selector from #more to .more. This is actually quite a common mistake that is the cause of many problems and questions asked here. The earlier you learn how to use id, the better.

Send SMTP email using System.Net.Mail via Exchange Online (Office 365)

Finally, Works!

Put smtpClient.UseDefaultCredentials = false; after smtpClient.Credentials = credentials; then problem resolved!

            SmtpClient smtpClient = new SmtpClient(smtpServerName);                          
            System.Net.NetworkCredential credentials = new System.Net.NetworkCredential(smtpUName, smtpUNamePwd);

            smtpClient.Credentials = credentials;
            smtpClient.UseDefaultCredentials = false;  <-- Set This Line After Credentials

            smtpClient.Send(mailMsg);
            smtpClient = null;
            mailMsg.Dispose();

Android: Pass data(extras) to a fragment

great answer by @Rarw. Try using a bundle to pass information from one fragment to another

What's the best way to add a full screen background image in React Native

The width and height with value null doesn't work for me, then I thought to use top, bottom, left, and right position and it worked. Example:

bg: {
        position: 'absolute',
        top: 0,
        bottom: 0,
        left: 0,
        right: 0,
        resizeMode: 'stretch',
},

And the JSX:

<Image style={styles.bg} source={{uri: 'IMAGE URI'}} />

Vue Js - Loop via v-for X times (in a range)

The same goes for v-for in range:

<li v-for="n in 20 " :key="n">{{n}}</li>

python pip: force install ignoring dependencies

pip has a --no-dependencies switch. You should use that.

For more information, run pip install -h, where you'll see this line:

--no-deps, --no-dependencies
                        Ignore package dependencies

How to copy folders to docker image from Dockerfile?

FROM openjdk:8-jdk-alpine RUN apk update && apk add wget openssl lsof procps curl RUN apk update RUN mkdir -p /apps/agent RUN mkdir -p /apps/lib ADD ./app/agent /apps/agent ADD ./app/lib /apps/lib ADD ./app/* /apps/app/ RUN ls -lrt /apps/app/ CMD sh /apps/app/launch.sh

by using DockerFile, I'm copying agent and lib directories to /apps/agent,/apps/lib directories and bunch of files to target.

How do you beta test an iphone app?

Creating ad-hoc distribution profiles

The instructions that Apple provides are here, but here is how I created a general provisioning profile that will work with multiple apps, and added a beta tester.

My setup:

  • Xcode 3.2.1
  • iPhone SDK 3.1.3

Before you get started, make sure that..

  • You can run the app on your own iPhone through Xcode.

Step A: Add devices to the Provisioning Portal

  1. Send an email to each beta tester with the following message:

    To get my app on onto your iPhone I need some information about your phone. Guess what, there is an app for that!

    Click on the below link and install and then run the app.

    http://itunes.apple.com/app/ad-hoc-helper/id285691333?mt=8

    This app will create an email. Please send it to me.

  2. Collect all the UDIDs from your testers.

  3. Go to the Provisioning Portal.

  4. Go to the section Devices.

  5. Click on the button Add Devices and add the devices previously collected.

Step B: Create a new provisioning profile

  1. Start the Mac OS utility program Keychain Access.

  2. In its main menu, select Keychain Access / Certificate Assistant / Request a Certificate From a Certificate Authority...

  3. The dialog that pops up should aready have your email and name it it.

  4. Select the radio button Saved to disk and Continue.

  5. Save the file to disk.

  6. Go back to the Provisioning Portal.

  7. Go to the section Certificates.

  8. Go to the tab Distribution.

  9. Click the button Request Certificate.

  10. Upload the file you created with Keychain Access: CertificateSigningRequest.certSigningRequest.

  11. Click the button Aprove.

  12. Refresh your browser until the status reads Issued.

  13. Click the Download button and save the file distribution_identify.cer.

  14. Doubleclick the file to add it to the Keychain.

  15. Backup the certificate by selecting its private key and the File / Export Items....

  16. Go back to the Provisioning Portal again.

  17. Go to the section Provisioning.

  18. Go to the tab Distribution.

  19. Click the button New Profile.

  20. Select the radio button Ad hoc.

  21. Enter a profile name, I named mine Evertsson Common Ad Hoc.

  22. Select the app id. I have a common app id to use for multiple apps: Evertsson Common.

  23. Select the devices, in my case my own and my tester's.

  24. Submit.

  25. Refresh the browser until the status field reads Active.

  26. Click the button Download and save the file to disk.

  27. Doubleclick the file to add it to Xcode.

Step C: Build the app for distribution

  1. Open your project in Xcode.

  2. Open the Project Info pane: In Groups & Files select the topmost item and press Cmd+I.

  3. Go to the tab Configuration.

  4. Select the configuration Release.

  5. Click the button Duplicate and name it Distribution.

  6. Close the Project Info pane.

  7. Open the Target Info pane: In Groups & Files expand Targets, select your target and press Cmd+I.

  8. Go to the tab Build.

  9. Select the Configuration named Distribution.

  10. Find the section Code Signing.

  11. Set the value of Code Signing Identity / Any iPhone OS Device to iPhone Distribution.

  12. Close the Target Info pane.

  13. In the main window select the Active Configuration to Distribution.

  14. Create a new file from the file template Code Signing / Entitlements.

  15. Name it Entitlements.plist.

  16. In this file, uncheck the checkbox get-task-allow.

  17. Bring up the Target Info pane, and find the section Code Signing again.

  18. After Code Signing Entitlements enter the file name Entitlements.plist.

  19. Save, clean, and build the project.

  20. In Groups & Files find the folder MyApp / Products and expand it.

  21. Right click the app and select Reveal in Finder.

  22. Zip the .app file and the .mobileprovision file and send the archive to your tester.

    Here is my app. To install it onto your phone:

    1. Unzip the archive file.

    2. Open iTunes.

    3. Drag both files into iTunes and drop them on the Library group.

    4. Sync your phone to install the app.

Done! Phew. This worked for me. So far I've only added one tester.

Get Selected value from dropdown using JavaScript

The first thing i noticed is that you have a semi colon just after your closing bracket for your if statement );

You should also try and clean up your if statement by declaring a variable for the answer separately.

function answers() {

var select = document.getElementById("mySelect");
var answer = select.options[select.selectedIndex].value;

    if(answer == "To measure time"){
        alert("Thats correct"); 
    }

}

http://jsfiddle.net/zpdEp/

How to check if a variable is an integer in JavaScript?

Just try this:

let number = 5;
if (Number.isInteger(number)) {
    //do something
}

How is using OnClickListener interface different via XML and Java code?

These are exactly the same. android:onClick was added in API level 4 to make it easier, more Javascript-web-like, and drive everything from the XML. What it does internally is add an OnClickListener on the Button, which calls your DoIt method.

Here is what using a android:onClick="DoIt" does internally:

Button button= (Button) findViewById(R.id.buttonId);
button.setOnClickListener(new View.OnClickListener() {
    @Override
    public void onClick(View v) {
        DoIt(v);
    }
});

The only thing you trade off by using android:onClick, as usual with XML configuration, is that it becomes a bit more difficult to add dynamic content (programatically, you could decide to add one listener or another depending on your variables). But this is easily defeated by adding your test within the DoIt method.

How to remove entity with ManyToMany relationship in JPA (and corresponding join table rows)?

As an alternative to JPA/Hibernate solutions : you could use a CASCADE DELETE clause in the database definition of your foregin key on your join table, such as (Oracle syntax) :

CONSTRAINT fk_to_group
     FOREIGN KEY (group_id)
     REFERENCES group (id)
     ON DELETE CASCADE

That way the DBMS itself automatically deletes the row that points to the group when you delete the group. And it works whether the delete is made from Hibernate/JPA, JDBC, manually in the DB or any other way.

the cascade delete feature is supported by all major DBMS (Oracle, MySQL, SQL Server, PostgreSQL).

There is an error in XML document (1, 41)

On a WEC7 project I'm working on, I got a similar error. The file I was serializing in was serialized out from an array of objects, so I figured the XML was fine. Also, I have had this working for a few previous classes, so it was quite a puzzle.

Then I noticed in my earlier work that every class that I was serializing/deserializing had a default constructor. That was missing in my failed case so I added it and and voila... it worked fine.

I seem to remember reading somewhere that this was required. I guess it is.

How can I call the 'base implementation' of an overridden virtual method?

Using the C# language constructs, you cannot explicitly call the base function from outside the scope of A or B. If you really need to do that, then there is a flaw in your design - i.e. that function shouldn't be virtual to begin with, or part of the base function should be extracted to a separate non-virtual function.

You can from inside B.X however call A.X

class B : A
{
  override void X() { 
    base.X();
    Console.WriteLine("y"); 
  }
}

But that's something else.

As Sasha Truf points out in this answer, you can do it through IL. You can probably also accomplish it through reflection, as mhand points out in the comments.

Excel VBA - Sum up a column

I have a label on my form receiving the sum of numbers from Column D in Sheet1. I am only interested in rows 2 to 50, you can use a row counter if your row count is dynamic. I have some blank entries as well in column D and they are ignored.

Me.lblRangeTotal = Application.WorksheetFunction.Sum(ThisWorkbook.Sheets("Sheet1").Range("D2:D50"))

C string append

strcpy(str1+strlen(str1), str2);

"The certificate chain was issued by an authority that is not trusted" when connecting DB in VM Role from Azure website

If you're seeing this error message when attempting to connect using SSMS, add TrustServerCertificate=True to the Additional Connection Parameters.

How to handle onchange event on input type=file in jQuery?

Try to use this:

HTML:

<input ID="fileUpload1" runat="server" type="file">

JavaScript:

$("#fileUpload1").on('change',function() {
    alert('Works!!');
});

?

receiving error: 'Error: SSL Error: SELF_SIGNED_CERT_IN_CHAIN' while using npm

Uninstall NPM and install it again.

As of February 27, 2014 npm no longer supports its self-signed certificates. http://blog.npmjs.org/post/78085451721/npms-self-signed-certificate-is-no-more

The link above suggests upgrading NPM using NPM. This also fails with SELF_SIGNED_CERT_IN_CHAIN...

How to configure heroku application DNS to Godaddy Domain?

I struggled a lot to resolve it Nothing seemed to work for me.

The steps I followed are mentioned here.

1 - Go to your App settings.

2 - Click on Add domain.

enter image description here

3 - A dialog will open & will ask you to enter the desired domain. (Please add it starting with www for instance - www.abcd.com )

enter image description here

4 - One added click on Next to move to the next dialog.

enter image description here

5 - After adding the domain you will get the DNS target, Now you need to navigate to GoDaddy and follow the following steps.

6 - Navigate to https://dcc.godaddy.com/domains & click on your domain.

7 - Once clicked you will navigate to https://dcc.godaddy.com/control/yourdomain/settings

8 - Scroll down to the bottom you will see Manage DNS.

enter image description here

9 - It will navigate you to DNS settings then add the entry similar to mentioned below and delete all other CNAME records. Here the value of points is your DNS target that you got in the 4th Step.

enter image description here

10 - Then after some time your site should be mapped to the Heroku app URL.

Set Font Color, Font Face and Font Size in PHPExcel

I recommend you start reading the documentation (4.6.18. Formatting cells). When applying a lot of formatting it's better to use applyFromArray() According to the documentation this method is also suppose to be faster when you're setting many style properties. There's an annex where you can find all the possible keys for this function.

This will work for you:

$phpExcel = new PHPExcel();

$styleArray = array(
    'font'  => array(
        'bold'  => true,
        'color' => array('rgb' => 'FF0000'),
        'size'  => 15,
        'name'  => 'Verdana'
    ));

$phpExcel->getActiveSheet()->getCell('A1')->setValue('Some text');
$phpExcel->getActiveSheet()->getStyle('A1')->applyFromArray($styleArray);

To apply font style to complete excel document:

 $styleArray = array(
   'font'  => array(
        'bold'  => true,
        'color' => array('rgb' => 'FF0000'),
        'size'  => 15,
        'name'  => 'Verdana'
    ));      
 $phpExcel->getDefaultStyle()
    ->applyFromArray($styleArray);

Get first line of a shell command's output

Yes, that is one way to get the first line of output from a command.

If the command outputs anything to standard error that you would like to capture in the same manner, you need to redirect the standard error of the command to the standard output stream:

utility 2>&1 | head -n 1

There are many other ways to capture the first line too, including sed 1q (quit after first line), sed -n 1p (only print first line, but read everything), awk 'FNR == 1' (only print first line, but again, read everything) etc.

Rotating x axis labels in R for barplot

You can simply pass your data frame into the following function:

rotate_x <- function(data, column_to_plot, labels_vec, rot_angle) {
    plt <- barplot(data[[column_to_plot]], col='steelblue', xaxt="n")
    text(plt, par("usr")[3], labels = labels_vec, srt = rot_angle, adj = c(1.1,1.1), xpd = TRUE, cex=0.6) 
}

Usage:

rotate_x(mtcars, 'mpg', row.names(mtcars), 45)

enter image description here

You can change the rotation angle of the labels as needed.

3-dimensional array in numpy

As much as people like to say "order doesn't matter its just convention" this breaks down when entering cross domain interfaces, IE transfer from C ordering to Fortran ordering or some other ordering scheme. There, precisely how your data is layed out and how shape is represented in numpy is very important.

By default, numpy uses C ordering, which means contiguous elements in memory are the elements stored in rows. You can also do FORTRAN ordering ("F"), this instead orders elements based on columns, indexing contiguous elements.

Numpy's shape further has its own order in which it displays the shape. In numpy, shape is largest stride first, ie, in a 3d vector, it would be the least contiguous dimension, Z, or pages, 3rd dim etc... So when executing:

np.zeros((2,3,4)).shape

you will get

(2,3,4)

which is actually (frames, rows, columns). doing np.zeros((2,2,3,4)).shape instead would mean (metaframs, frames, rows, columns). This makes more sense when you think of creating multidimensional arrays in C like langauges. For C++, creating a non contiguously defined 4D array results in an array [ of arrays [ of arrays [ of elements ]]]. This forces you to de reference the first array that holds all the other arrays (4th dimension) then the same all the way down (3rd, 2nd, 1st) resulting in syntax like:

double element = array4d[w][z][y][x];

In fortran, this indexed ordering is reversed (x is instead first array4d[x][y][z][w]), most contiguous to least contiguous and in matlab, it gets all weird.

Matlab tried to preserve both mathematical default ordering (row, column) but also use column major internally for libraries, and not follow C convention of dimensional ordering. In matlab, you order this way:

double element = array4d[y][x][z][w];

which deifies all convention and creates weird situations where you are sometimes indexing as if row ordered and sometimes column ordered (such as with matrix creation).

In reality, Matlab is the unintuitive one, not Numpy.

sendKeys() in Selenium web driver

I believe Selenium now uses Key.TAB instead of Keys.TAB.